Sie sind auf Seite 1von 55

1.

Consider the following statements about Paika rebellion:

1. Paiks of Odisha were the traditional landed militia.


2. BakshiJagabandhuBidyadhar, the Raja of Khurda, led the rebellion against the Company.
3. The rebels were not supported by zamindars of the region.

Which of the above statements is/are correct?

(a) 1 only
(b) 1 and 2 only
(c) 2 and 3 only
(d) 1, 2 and 3

Solution: (a)

The Paiks of Odisha were the traditional landed militia (‗foot soldiers‘ literally) and enjoyed
rent free land tenures for their military service and policing functions on a hereditary basis.
Hence Statement 1 is correct. The English Company‘s conquest of Odisha in 1803, and the
dethronement of the Raja of Khurda had greatly reduced the power and prestige of the Paiks.
Further, the extortionist land revenue policy of the Company caused resentment among
zamindars and peasants alike.
BakshiJagabandhuBidyadhar had been the military chief of the forces of the Raja of Khurda.
Hence Statement 2 is incorrect. With active support of Mukunda Deva, the last Raja of
Khurda, and other zamindars of the region, BakshiJagabandhuBidyadhar led a sundry army
of Paikas forcing the East India Company forces to retreat for a time. Hence Statement 3 is
incorrect. The rebellion came to be known as the PaikaBidroh (rebellion). The initial
success of the rebels set the whole province covering Odisha in arms against the British
government for some time. Though Khurda was back under Company control by mid-1817,
the Paika rebels resorted to guerilla tactics. The rebellion was brutally repressed by 1818.

2. Consider the following pairs:

Tribal Revolts Year

1. Kol rebellion 1931


2. Warli revolt 1940
3.Bastar rebellion 1910

Which of the pairs given above is/are correct?


(a) 1 only
(b) 1 and 2
(c) 3 only
(d) 2 and 3

Solution: d)
Tribal Revolts Year
1. Kol rebellion 1831
2. Bastar rebellion 1910
3. Warli revolt 1940
3. Consider the following statements:

1. Mappilas were the Muslim tenants inhabiting the Malabar region.


2. Mappila rebellion merged with the Civil Disobedience Movement.

Which of the statements given above is/are correct?


(a) 1 only
(b) 2 only
(c) Both 1 and 2
(d) Neither 1 nor 2

Solution: (a)
Hike in revenue demand and reduction of field size, coupled with the oppression of officials,
resulted in widespread peasant unrest among the Moplahs of Malabar. Twenty-two rebellions
took place between 1836 and 1854. None, however, proved successful.
The Mappilas were the Muslim tenants inhabiting the Malabar region where most of the
landlords were Hindus. Hence Statement 1 is correct.
Their grievances centred around lack of security of tenure, high rents, renewal fees and other
oppressive exactions. The Mappila tenants were particularly encouraged by the demand of
the local Congress body for a government legislation regulating tenant-landlord relations.
Soon, the Mappila movement merged with the ongoing Khilafat agitation.
. What began as an antigovernment and anti-landlord affair acquired communal overtones.
The communalisation of the rebellion completed the isolation of the Mappilas from the
Khilafat-Non- Cooperation Movement. By December 1921, all resistance had come to a stop.
Hence Statement 2 is incorrect.

4. Which of the following was not a reason for the British to annex Nepal?
(a) Extend commercial links with Central Asia through Himalayas.
(b) To manage Border issues
(c) Check expansion of the French in Central Asia.
(d) Gain the pleasant hill stations in the Himalayas for the British officials for recreation.

Answer: (c)

Explanation: the aggressive expansion policy of the British with the neighbours led to a series
of war at the cost of Indian soldiers and taxes but not for their interest. The first war with
Nepal took place in 1814. As the outcome of the British victory the districts of Gharwal and
Kumoan was ceded to India. The kingdom of Nepal withdrew from Sikkim. The advantages
were that now British Empire could extend trade with central Asia. Hence (a) is correct.
However the French was not expanding in Central Asia. It was the objective of British to
annex Burma to check the French expansion in south-east Asia. Hence option (c) is
incorrect.
Border policy issues and the need for hill stations were some of the other reasons.
Source: Chapter 7 - Administrative changes after 1858, Modern India, NCERT, pg. 133.

5. With reference to the Raja Rammohan Roy, which of the following statements is/are
correct?
1. He supported the concept of one God, propounded by Upanishads.
2. He quoted Vedas to support his viewpoints.

Select the correct answer using the code given below.


(a) 1 only
(b) 2 only
(c) Both 1 and 2
(d) Neither 1 nor 2

Answer: (c)
Explanation: Though Rammohan Roy rejected the infallibility of the Vedas, he supposed his
viewpoints by using quotations from Vedas. He also accepted the concept of one God of
Upanishads. His God was shapeless, invisible, omnipresent, omnipotent and the guiding spirit
of the Universe. Both the statements are correct.
Reference: Page: 275; A new look at modern Indian history, B.L.Grover and Alka Mehta

6. With reference to Raja Rammohan Roy, which of the following statements is/are
correct?

1. He was the founder of Vedanta college.


2. He supported the traditional Zamindars against the British rule.
3. He renounced Hindu religion and its philosophy.

Select the correct answer using the code given below.


(a) 1 only
(b) 1 and 2 only
(c) 1 and 3 only
(d) 1, 2 and 3

Answer: (a)
Explanation: He established the vedanta college in 1825, in which courses in both Indian
learning and Western sciences were offered. Statement 1 is correct.
He condemned the oppressive practices of Bengal zamindars and demanded that the rents
paid by the actual cultivators should be permanently fixed like the Permanent Settlement of
1793. Statement 2 is incorrect.
He vigorously defended the Hindu religion and philosophy from the ignorant attack of the
Christian missionaries. He also accepted the concept of one God of Upanishads. His God was
shapeless, invisible, omnipresent, omnipotent and the guiding spirit of the Universe.
Statement 3 is incorrect.
Reference: Page: 130, 131, 132; History of Modern India, Bipan Chandra

7. With reference to the Theosophical Society, which of the following statements is/are
correct?

1. The organisation was associated with Hindu Renaissance.


2. It accepted the doctrine of Karma and the transmigration of soul.
Select the correct answer using the code given below.
(a) 1 only
(b) 2 only
(c) Both 1 and 2
(d) Neither 1 nor 2

Answer: (c)
Explanation: Theosophical Society was founded by westerners who drew inspiration from
Indian culture. It came to be allied with Hindu Renaissance. The society accepted Hindu
beliefs in re-incarnation, karma and was inspired by Upanishads, Yoga and Vedanta. Both
the statements are correct.
Reference: Page: 279; A new look at modern Indian history, B.L.Grover and Alka Mehta
Page: 225; History of Modern India, Bipan Chandra

8. With reference to Annie Besant, which of the following statements is/are correct?

1. She was associated with the foundation of Benaras Hindu University.


2. She was a leader of Home Rule League movement.
3. She was associated with women‘s education.

Select the correct answer using the code given below.


(a) 3 only
(b) 2 only
(c) 2 and 3
(d) 1, 2 and 3

Answer: (d)
Explanation: Annie Besant laid the foundation of Central Hindu School at Benaras in 1898
which later became the Benares Hindu University in 1916.
In 1916, two Home Rule League movements were started under the leadership of Bal
Gangadhar Tilak and Annie Besant. Tilak's movement was concentrated in Maharashtra
(except Bombay city), Karnataka, Central Provinces and Berar while Besant's movement
covered the rest of India.
She also did much for the cause of female education. All Statements are correct.
Reference: Page: 279; A new look at modern Indian history, B.L.Grover and Alka Mehta
Page: 226, 271; History of Modern India, Bipan Chandra
Page:161, 162; India's Struggle for Independence, Bipan Chandra and others

9. With reference to Ishwar Chandra Vidyasagar, which of the following statements


is/are correct?

1. He was a pioneer of women education.


2. He advocated widow remarriage.
3. He was responsible for opening of Sanskrit studies to non-Brahmin students.

Select the correct answer using the code given below.


(a) 2 only
(b) 2 and 3 only
(c) 1 and 2 only
(d) 1, 2 and 3

Answer: (d)
Explanation: Ishwar Chandra Vidyasagar was deeply interested in the education of women.
As Government Inspector of schools, he organised 35 girls' schools and ran many of them at
his own expense. Bethune school was symbol of a powerful movement of women's education
and he was Secretary in that school. Vidyasagar served as the Principal of Sanskrit College of
Calcutta in 1951. He opened up the study of Sanskrit scriptures to non-Brahmin students. He
led a powerful movement in favour of widow remarriage.
Reference: Page: 136, 137; History of Modern India, Bipan Chandra

10. Match the following organisations with the regions associated with them.

1. Deoband movement a. Bengal


2. TattvabodhiniSabha b. Punjab
3. Wahabi movement c. Bombay
4. RehnumaiMazdayasanSabha d. United Province

Select the correct answer using the code given below.

(a) 1-d, 2-a, 3-b, 4-c


(b) 1-b, 2-d, 3-c, 4-a
(c) 1-d, 2-c, 3-a, 4-b
(d) 1-b, 2-d, 3-a, 4-c

Answer: (a)
Explanation: Muhammad QasimWanotavi and Rashid AhamadGangohi were the leaders of
the Deoband school in Saharanpur, UP. It was under the leadership of orthodox Muslim
ulema and was associated with the preaching of Islamic faith and training leaders for the
Muslim community. It shut out English culture and Western education.
TattvabodhiniSabha was founded by Debendranath Tagore in 1839. He was attracted to the
ideas of Raja Rammohan Roy and joined the Brahmo Samaj in 1842.
Shah Walliullah founded the Wahabi movement as a revivalist movement. He wanted to
restore the purity of Islam and aimed at creating a homeland for the Muslims. An armed
campaign was led against the Sikhs in Punjab and later against the British.
RehnumaiMazdayasanSabha (Religious Reform Association) was set up by educated Parsis
in Bombay to bring about reforms in their community. The newspaper Rast-Gofter(Truth-
Teller) was associated with it. It focussed on social reforms in Parsi community such as
removal of Purdah system, raising the age of marriage and women education.
NaorojiFurdonji, S.S.Bengalee, DadabhaiNaoroji were associated with it.
Reference: Page: 228; History of Modern India, Bipan Chandra
Page: 275, 280, 281; A new look at modern Indian history, B.L.Grover and Alka Mehta
11. With reference to Surya Sen, which of the following statements is/are correct?

1. He was the head of the Indian Republican Army.


2. He was responsible for the armed raid of Government treasury and armoury in
Chittagong.

Select the correct answer using the code given below.


(a) 1 only
(b) 2 only
(c) Both 1 and 2
(d) Neither 1 nor 2

Answer: (c)
Explanation: Surya Sen was an active participant of the Non Co-operation movement. He
participated in revolutionary activity after that phase. He was also the Secretary of Chittagong
district Congress committee. He led the Chittagong group. He, along with his associates,
conducted raids on the armouries in 1930, under the banner of Indian Republican Army. They
destroyed telephone and telegraph lines connecting Chittagong with the rest of Bengal. He
was finally arrested and hanged in 1934.
Both Statements are correct.
Reference: Page: 251, 252; India's Struggle for Independence, Bipan Chandra and others
Page: 300; History of Modern India, Bipan Chandra

12. With reference to Nehru Report, which of the following statements is/are correct?

1. It rejected the principle of separate communal electorates.


2. It provided for reservation for Muslims in all provinces.
3. It advocated universal adult suffrage.

Select the correct answer using the code given below.


(a) 1 and 3
(b) 3 only
(c) 1 and 2
(d) 1, 2 and 3

Answer: (a)
Explanation: The report defined Dominion Status as the desirable form of govt. It rejected
communal electorates. It envisaged joint electorates with reservation of seats for minorities
(Muslims). This reservation would be available only in provinces where the Muslims are in
minority and not in Punjab and Bengal. These clauses were rejected by the Muslim League.
Statement 1 is correct. Statement 2 is incorrect.
The report also recommended universal adult suffrage, equal rights for women, freedom to
form unions, dissociation of state from religion.Statement 3 is correct.
Reference: Page: 263; India's Struggle for Independence, Bipan Chandra and others

13. With reference to the Civil Disobedience Movement, which of the following
statements is/are correct?
1. Rani Gaidilieuspearheaded the movement in Mizoram.
2. There were incidents where the army showed support to the movement.

Select the correct answer using the code given below.


(a) 1 only
(b) 2 only
(c) Both 1 and 2
(d) Neither 1 nor 2

Answer: (b)
Explanation: The young Rani Gaidilieu raised the banner of rebellion in Nagaland against
foreign rule. She was arrested and sentenced to life imprisonment. She was released only in
1947. Statement 1 is incorrect.
In Peshawar, two platoons of Garhwali soldiers refused to open fire on non-violent
demonstrators though it meant court martial. Statement 2 is correct.
Both Statements are correct.
Reference: Page: 305; History of Modern India, Bipan Chandra

14. With reference to the Civil Disobedience Movement, which of the following
statements is/are correct?

1. The United Provinces had a 'no-rent, no-revenue' campaign against the local
zamindars.
2. Eastern India had a campaign against the Chowkidara Tax.

Select the correct answer using the code given below.


(a) 1 only
(b) 2 only
(c) Both 1 and 2
(d) Neither 1 nor 2

Answer: (c)
Explanation: Forest laws were broken in Maharashtra, Karnataka and the Central provinces.
In United Provinces, the movement became a campaign of 'no-rent, no-revenue'. The tenants
ought not to pay the rent to the Zamindars, who in turn ought not to pay revenue to the govt.
As the Zamindars were loyal to the govt, it became a no rent campaign. Eastern India had a
campaign against Chowkidara tax which paid the police force at rural level. It spread through
Bihar and Bengal.
Both Statements are correct.
Reference: Page: 280, 281; India's Struggle for Independence, Bipan Chandra and others

15. With reference to the Gandhi-Irwin Pact, which of the following statements is/are
correct?
1. The pact called for the commutation of death sentence of Bhagat Singh and other
leaders.
2. The pact was negotiated to enable Congress to participate in the first Round Table
Conference.

Select the correct answer using the code given below.


(a) 1 only
(b) 2 only
(c) Both 1 and 2
(d) Neither 1 nor 2

Answer: (d)
Explanation: Congress leaders were imprisoned during the Civil Disobedience Movement
and the first RTC (Round Table Conference) happened without them. They were released as
part of Gandhi-Irwin pact. They participated in the second RTC but the British govt refused
to accept the Dominion status demand. Gandhi refused to make the Pact conditional on the
commutation of the death sentences of Bhagatsingh and his comrades. Both Statements are
incorrect.
Reference: Page: 280, 281; India's Struggle for Independence, Bipan Chandra and others
Page: 307; History of Modern India, Bipan Chandra

16. With reference to the Government of India Act of 1935, which of the following
statements is/are correct?

1. It provided for universal adult franchise by expanding the voters list


2. Defence and foreign affairs were vested with the Federal legislature but the Viceroy
had special powers of veto to overrule them.

Select the correct answer using the code given below.


(a) 1 only
(b) 2 only
(c) Both 1 and 2
(d) Neither 1 nor 2

Answer: (d)
Explanation: The electorate was restricted to around one-sixth of the adults. It was based on
property and few other parameters. Statement 1 is incorrect.
Defence and foreign affairs were outside the control of the Federal legislature. The Viceroy
would retain special control over other subjects. Statement 2 is incorrect.
Reference: Page: 317; India's Struggle for Independence, Bipan Chandra and others
Page: 309; History of Modern India, Bipan Chandra

17. With reference to the Tebagha struggle, which of the following statements is/are
correct?
1. It was a tribal movement in the northeast.
2. The movement was focussed on the issues of sharing and storage of the harvested
crops.

Select the correct answer using the code given below.


(a) 1 only
(b) 2 only
(c) Both 1 and 2
(d) Neither 1 nor 2

Answer: (b)
Explanation:Tebagha movement was a peasant movement which took place in Bengal. In
1946, the share-croppers refused to pay the half share of crop as rent to the jotedars but only
one-third. Also the crop would be stored in their own godowns and not that of the jotedars.
The Bengal ministry headed by Suhrawardy published the Bengal Bargadars Temporary
Regulation Bill in the Gazette but the bill was passed in 1950.
Statement 1 is incorrect. Statement 2 is correct.
Reference: Page: 352, 353; India's Struggle for Independence, Bipan Chandra and others

18. With reference to the peasant movements, match the following.

1. Kerala a. Hali system


2. Maharashtra b. KarshakaSangham
3. Gujarat c. Malguzari system

Select the correct answer using the code given below.


(a) 1-a, 2-c, 3-b
(b) 1-b, 2-c, 3-a
(c) 1-c, 2-b, 3-a
(d) 1-b, 2-a, 3-c

Answer: (b)
Explanation:KarshakaSanghams were peasant associations formed in the Kerala region,
demanding abolition of feudal levies and protection of tenants.
Hali system was a bonded labour system in Gujarat, in which the depressed class peasants
were hereditary labourers under the upper class land owners.
Malguzari or Mahalwari is a revenue settlement found in several parts of Maharashtra,
Madras and Punjab during British rule.
Reference: Page: 346, 351; India's Struggle for Independence, Bipan Chandra and others

19. With reference to the Cripps mission, which of the following statements is/are incorrect?
1. It offered to accept the demand for Dominion status.
2. It did not talk about any immediate transfer of power.
Select the correct answer using the code given below.
(a) 1 only
(b) 2 only
(c) Both 1 and 2
(d) Neither 1 nor 2
Answer: (d)
Explanation: Cripps mission was sent under Sir Stafford Cripps in March 1942. The terms
of the proposal included the following:
(i) The demand for Dominion status was accepted.
(ii) A constitution making body would be formed after the war. It would include elected
members from provincial assemblies and nominated members from the princely states.
(iii) Any province which was not prepared to accept the Constitution, can sign a separate
agreement with Britain. This accommodated the demand for Pakistan.
Both statements are correct.
Reference: Page: 455; India's Struggle for Independence, Bipan Chandra and others

20. With reference to the Quit India movement, which of the following statements is/are
correct?

1. The delay in arresting the Congress leadership gave the immediate impetus to the
movement.
2. Gandhi refused to condemn the violence of the people during the movement.
3. During the course of the movement, 'parallel governments' were formed in many parts
of the country.

Select the correct answer using the code given below.


(a) 1 only
(b) 1 and 3 only
(c) 2 and 3 only
(d) 1, 2 and 3

Answer: (c)
Explanation: The Quit India resolution was launched by Gandhi on 8 Aug 1942. In the early
hours of 9 August, all the top leaders of INC were arrested and taken to unknown locations.
Statement 1 is incorrect.
During the course of the movement, people resorted to acts of violence such as sabotaging
railway tracks, destruction of telephone and telegraph wires,attacking railway stations and
police stations. The govt pressed Gandhi to condemn the violence. But he refused and instead
held the govt responsible for it. Statement 2 is correct.
'Parallel governments' were formed in parts of Bengal, U.P, Maharashtra and several other
provinces. The govt offices were disabled by attacks. Statement 3 is correct.
Reference: Page: 460, 464, 466; India's Struggle for Independence, Bipan Chandra and
others
Page: 323; History of Modern India, Bipan Chandra

21. With respect to the Cabinet Mission, which of the following are its provisions?
1. Only Defence, Communication and Foreign Affairs would be controlled by the
federal government.
2. It did not provide for partition of the country.

Select the correct answer using the code given below.


(a) 1 only
(b) 2 only
(c) Both 1 and 2
(d) Neither 1 nor 2

Answer: (c)
Explanation: Cabinet Mission was sent on March 1946 with the following terms:
(i) Maintaining national unity while conceding the largest measure of regional autonomy
(ii) Two tier federal plan
(iii) Only Defence, Communication and Foreign Affairs would be controlled by the federal
government
(iv) Provinces could form regional unions.
(v) Existing provinces were divided into three groups.
Group A - Hindu majority (Madras, Bombay, Central Provinces, United Provinces, Bihar,
Odisha, Delhi, Coorg, Ajmer-Marwara)
Group B - Muslim majority (Punjab, NorthWest Frontier Provinc, Sind, Baluchistan)
Group C - Bengal, Assam
(vi) It endorsed the view that a minority cannot be allowed to place a veto on the advance of
the majority. It did not provide for partition.
Both statements are correct.
Reference: Page: 492; India's Struggle for Independence, Bipan Chandra and others
Page: 327; History of Modern India, Bipan Chandra

22. Consider the following pairs:


INC Session Importance in Congress
1. 1924 Belgaum Mahatma Gandhi became President
2. 1889 Calcutta Sarojini Naidu became 1st women President
3. 1888 Allahabad Wedderburn became first English President.
Which of the pairs given above is/are correct?
(a) 1 only
(b) 1 and 3
(c) 3 only
(d) 2 and 3
Solution: (a)
1885 Bombay- 72 delegates attended the session.
1887 Madras- Badruddin Tyabji became first Muslim President.
1888 Allahabad- George Yule became first English President.
1898 Madras- Social reform was set as the main goal.
1907 Surat- Congress split.
1908 Madras- Constitution for the Congress formed.
1916 Lucknow- Congress merged. Pact with Muslim League.
1917 Calcutta- Annie Besant became 1st women President.
1920 Nagpur- Gandhian programme was adopted. Change in congress constitution.
1922 Gaya- Formation of Swaraj Party.
1924 Belgaum- Gandhi became President (Gandhi became President here first and last time).
1925 Kanpur- Sarojini Naidu became 1st Indian women President.

23. Consider the following statements:


1. Neel Darpan is a play about the plight of indigo farmers written by DinbandhuMitra.
2. Michael Madhusudan Dutt translated the play into Hindi
3. The Government‘s response to the Indigo Revolt was similar to its response towards civil
rebellions and tribal uprisings during the period.
Which of the above statements is/are correct?
(a) 1 only
(b) 2 and 3 only
(c) 1 and 3 only
(d) 1, 2 and 3
Solution: (a)
DinabandhuMitra is primarily known for his play about the plight of indigo farmers Neel
Darpan. Hence Statement 1 is correct.
The Indigo Revolt (1858) or Nilbidraha in Bengali was the revolt of the indigo farmers
against the indigo planters. It was just one year after the Sepoy Revolt. Mitra stormed the
social and the literary circle of Bengal by his most notable play Neel darpan in the year 1860.
His first hand experience of the indigo cultivators, while on the job as the post master in rural
Orissa and Bengal, were reflected in the drama.
Michael Madhusudan Dutt translated the play into English immediately after it was
published. Hence Statement 2 is incorrect.
The Government‘s response to the Revolt was rather restrained and not as harsh as in the case
of civil rebellions and tribal uprisings. Government appointed a commission to inquire into
the problem of indigo cultivation. Hence Statement 3 is incorrect. Evidence brought before
the Indigo Commission and its final report exposed the coercion and corruption underlying
the entire system of indigo cultivation. The result was the mitigation of the worst abuses of
the system. The Government issued a notification in November 1860 that ryotscould not be
compelled to sow indigo and that it would ensure that all disputes were settled by legal
means. But the planters were already closing down the factories they felt that they could not
make their enterprises pay without the use of force and fraud.

24. Consider the following statements about BalagangadharaTilak:


1. He started the practice of using the traditional religious Ganapati festival to propagate
nationalist ideas.
2. He started the Shivaji festival to stimulate nationalism among young Maharashtrians.
3. He initiated a no-tax Campaign in Maharashtra during 1896-97 with the help of the
Servants of India Society.
4. He helped the Chapekar brothers in the murder of ICS official Rand .
Which of the statements given above is/are correct?
(a) 1, 2 and 3
(b) 1 and 2
(c) 2 and 4
(d) 1, 2 and 4
Solution: (b)
In 1893, Tilak started the practice of using the traditional religious Ganapati festival to
propagate nationalist ideas through patriotic songs and speeches. Hence Statement 1 is
correct.
In 1896, he started the Shivaji festival to stimulate nationalism among young
Maharashtrians. Hence Statement 2 is correct. In the same year, he organized an all-
Maharashtra campaign for the boycott of foreign cloth in protest against the imposition of the
excise duty on cotton.
BalgangadharTilak was, perhaps the first among the national leaders to grasp the important
role that the lower middle classes, peasants, artisans and workers could play in the national
movement and, therefore, he saw the necessity of bringing them into the Congress fold. He
criticized the Congress for ignoring the peasant and wanted to bring them into the Congress
fold. In pursuance of this objective, he initiated a no-tax Campaign in Maharashtra during
1896-97 with the help of the young workers of the Poona SarvajanikSabha. Hence Statement
3 is incorrect.
Referring to the official famine code whose copies he got printed in Marathi and distributed
by the thousand, he asked the famine-stricken peasants of Maharashtra to withhold payment
of land revenue if their crops had failed.
Tilak was by 1897 well-known in Maharashtra, both as a militant nationalist and as a hostile
arid effective journalist. The Government was looking for an opportunity to make an example
of him. The Rand murder gave them the opportunity. The British-owned Press and the
bureaucracy were quick to portray the Rand murder as a conspiracy by the Poona Brahmins
led by Tilak.
The Government investigated the possibility of directly involving Tilak in Rand‘s
assassination. But no proof could be found. Moreover, Tilak had condemned the
assassination describing it as the horrible work of a fanatic. Hence Statement 4 is incorrect.

25.„The country‟s emancipation can only be achieved by removing the clouds of


lethargy and indifference which have been hanging over the peasant, the soul of India.
We must remove these clouds, and for that we must completely identify ourselves with
the peasant -we must feel that he is ours and we are his.‟
The above lines can be attributed to which nationalist leader?
(a) G K Gokhale
(b) Mahatma Gandhi
(c) DadabhaiNaoroji
(d) BalgangadharTilak
Solution: (d)
BalgangadharTilak was, perhaps the first among the national leaders to grasp the important
role that the lower middle classes, peasants, artisans and workers could play in the national
movement and, therefore, he saw the necessity of bringing them into the Congress fold.
Criticizing the Congress for ignoring the peasant, he wrote in the Kesariin early 1897:
‗The country‘s emancipation can only be achieved by removing the clouds of lethargy and
indifference which have been hanging over the peasant, who is the soul of India. We must
remove these clouds, and for that we must completely identify ourselves with the peasant- we
must feel that he is ours and we are his.‘ Only when this is done would ‗the Government
realize that to despise the Congress is to despise the Indian Nation. Then only will the efforts
of the Congress leaders be crowned with success.‘

26. Consider the following statements:


1. Section 124A of the Indian Penal Code said that whoever attempted to excite feelings of
disaffection against the Government could be punished
2. Indian journalists fearing punishment by law often complied with the government of the
day
Which of the statements given above is/are correct?
(a) 1 only
(b) 2 only
(c) Both 1 and 2
(d) Neither 1 nor 2
Solution: (a)
To arouse political consciousness, to inculcate nationalism, to expose colonial rule, to ‗preach
disloyalty‘ was no easy task, for there had existed since 1870 Section 124A of the Indian
Penal Code according to which ‗whoever attempts to excite feelings of disaffection to the
Government established by law in British India‘ was to be punished with transportation for
life or for any term or with imprisonment upto three years. This clause was, moreover, later
supplemented with even more strident measures. Hence Statement 1 is correct.
Indian journalists adopted several clever stratagems and evolved a distinctive style of writing
to remain outside the reach of the law. Hence Statement 2 is incorrect. Since Section 124A
excluded writings of persons whose loyalty to the Government was undoubted, they
invariably prefaced their vitriolic writing with effusive sentiments of loyalty to the
Government and the Queen. Another strategem was to publish anti-imperialist extracts from
London-based socialist and Irish newspapers or letters from radical British citizens knowing
that the Indian Government could not discriminate against Indians by taking action against
them without touching the offending Britishers. Sometimes the extract from the British
newspaper would be taken without quotation marks and acknowledgement of the source, thus
teasing the British-Indian bureaucracy into contemplating or taking action which would have
to be given up once the real source of the comment became known. For example, a
sympathetic treatment of the Russian terrorist activities against Tsarism would be published
in such a way that the reader would immediately draw a parallel between the Indian
Government and the Revolutionary Terrorists of Bengal and Maharashtra.
Often the radical expose would take the form of advice and warning to the Government as if
from a well-wisher, as if the writer‘s main purpose was to save the authorities from their own
follies. B.G. Tilak and Motilal Ghosh were experts at this form of writing. Some of the more
daring writers took recourse to irony, sarcasm, banter, mock-seriousness and burlesque.
27. Consider the following pairs:
Newspaper Editor
1. Swadesamitran G. SubramaniyaIyer
2. Indian Mirror Dadabhai Naoroji
3. Advocate G P Varma
Which of the pairs given above is/are correct?
(a) 1 only
(b) 1 and 3
(c) 3 only
(d) 2 and 3
Solution: (b)
The Hindu and Swadesamitranunder the editorship of G. SubramaniyaIyer,
Indian Mirror under N.N. Sen,
Voice of India under Dadabhai Naoroji,
Hindustani and Advocate under G.P. Varma

28. Consider the following pairs:


1. Alipore conspiracy - PrafullaChaki
2. Delhi conspiracy - Sachin Sanyal and Rasbihari Gosh
3. Nasik Conspiracy - V.D. Sarvarkar
4. Kakori conspiracy - RamprasadBismal and Ashfatulalh Khan
Which of the above pairs is/are correctly matched?
(a) 1,2 and 3
(b) 2,3 and 4
(c) 2 and 4 only
(d) All of the above

Answer: (d)
Explanation: In 1908 a revolutionary conspiracy was intrigued to kill the Chief Presidency
Magistrate D.H. Kingford of Muzaffarpur. The task was entrusted to Khudiram Bose and
PrafullaChaki. They threw the bombs on a vehicle coming out of the magistrate‘s home on
April 30, 1908.
Delhi-Lahore Conspiracy was organised by the Indian revolutionary underground in Bengal
and Punjab and headed by Rashbehari Bose to assassinate the then Viceroy of India, Lord
Hardinge. Basant Kumar Biswas, Amir Chand and AvadhBehari were convicted and
executed in the trial of this Delhi Conspiracy Case.
On December 21, 1909, AnantLaxmanKanhere (one of the member of Abhinav Bharat
Society) shot the Collector of Nashik , A M T Jackson in the theatre at Nasik. This
sensational murder is known as Nasik Conspiracy Case. The members of the Abhinav Bharat
Society viz. V.D. Savarkar, Ganesh Savarkar, etc. were convicted and punished.
Kakori train robbery or Kakori Case was a train robbery against the British Indian
Government.It was organised by revolutionary organisation i.e., Hindustan Republican
Association (HRA) under the leadership of Ram Prasad Bismil and supported by Ashfaqulla
Khan, RajendraLahiri, Chandrashekhar Azad.
29.Which of the Following are Correctly matched?

1. Hindu-Musalmanki Jai- Khilafat Movement


2. Champaran Satyagraha-First Civil Disobedience
3. Ahmedabad Mill Strike-First Hunger Strike
4. Kheda Satyagraha-First Non Cooperation
Choose the answer using the code given below
a) 1 and 2
b) 2 and 3
c) 1, 2 and 3
d) 1, 2, 3 and 4

Ans-d

EXPLANATION
Mahatma Gandhi had a central role in the juxtaposition of the Khilafat Movement with
India‘s freedom struggle, primarily the Non Cooperation Movement. Mahatma Gandhi had
already materialized his ‗Satyagraha‘, the non-violent civil resistant nationalist movement
mostly on account of the various inhuman acts of repression by the British Government such
as the Rowlatt Act of 1919, the imposition of Marshall Law in Punjab in 1919 and
subsequently, the JalianwallahBahg Massacre of Aril 1919 among the others. Mahatma
Gandhi saw the Khilafat Movement as a brilliant opportunity to bring together the Hindus
and the Muslims and their respective causes against one common authority of exploitation
and domination.

One of the most important instances in India‘s freedom struggle was provided by the
Khilafat movement in terms of the unity among the Hindus and the Muslims. This was
mostly on account of the intertwining of the leaders of the Indian National Congress and the
Khilafat Movement themselves. This campaign saw the popularization of the slogan ‘Hindu-
Musalmaanki Jai’ during strikes, protests and demonstrations across the country. The
scenario of the Hindu-Muslim consonance went in tune with Mahatma Gandhi‘s idea that
freedom from the British Raj could only be achieved if the Hindus and the Muslims both
worked together and collectively fought for their freedom.

The Champaran Satyagraha is considered to be a vital event in the history of India‘s freedom
struggle. It is consideresas India‘s First Civil Disobedience movement launched by
Mahatma Gandhi to protest against the injustice meted out to tenant farmers in Champaran
district of Bihar.
In the Ahmedabad Mill strike Gandhi intervened in a dispute between the mill owners of
Ahmedabad and the workers over the issue of discontinuation of plague bonus. He undertook
a fast unto death to strengthen the workers resolve and at the same time pressurize the mill
owners.

In the Kheda Satyagraha , which is considered as the first Non Cooperation Gandhi
organised a movement to support peasants of the Kheda district. People of Kheda were
unable to pay the high taxes levied by the British due to crop failure and a plague epidemic.
He supported the peasants cause and asked them to withhold revenue.

30. What were the causes behind the Nationalist resurgence in India towards the end of
the First World War-
1. Inflation and fall in economic activity.
2. Infusion of Foreign Capital
3. October Revolution
Choose the correct answer using the code given below
a) 1, 2 and 3
b) 1 and 2
c) 2 and 3
d) 3 only
Ans-a
EXPLANATION
Towards the end of the first world war there was a resurgence of nationalist activities in
India and in many other colonies in Asia and Africa.
The main factors were-
a)Economic Hardships-
i)The economic situation in the post war years had taken a turn for the worse. There was first
a rise in prices and then a depression in economic activity. Indian Industries , which had
prospered during the war because foreign imports of manufactured goods had ceased , faced
losses and closure.
ii)Foreign Capital began to be invested in India on a large scale .The Indian industrialist
wanted protection of their industries through imposition of high customs duties and grant of
government aid. They realized that a strong nationalist movement and an independent Indian
government alone could secure these.
III)The workers and artisans ,facing unemployment and high prices also turned actively
towards the nationalist movement .
IV)On one hand the Peasantry faced deepening poverty and high taxation on the other hand
the educated urban faced increasing unemployment .
b)International Situation-
I)The First world war gave tremendous impetus to nationalism all over Asia and Africa. In
order to win popular support the Allied Nations (Britain ,United Nations, France, Italy and
Japan ) promised a new era of democracy. However at the Paris Peace Conference and in the
different peace settlements ,all the war time promises were forgotten .
II)Russian Revolution (7th Nov 1917) -The Bolshevik Party of workers overthrew the czarist
regime and founded the first socialist state ,thet of Soviet Union ,under the leadership of VI
Lenin . The Soviet Union unilaterally renounced the czarist imperialist rights in China and
the rest of Asia , gave rights of self determination to former czarist colonies in Asia and gave
equal status to the Asian nationalities within its borders.
The October revolution brought home the message that immense power lay with the people
and the masses were capable of challenging the mightiest of tyrants provided they were
organized ,united and determined.
31. Consider the following statements regarding the Education policy of 1913:
1. The bill on compulsory primary education was first moved by Gopal Krishna Gokhale
in 1911.
2. The government resolution on Education policy 1913 was focused on Primary,
secondary, higher and women education.
3. The government promised to extend grants to the education department.
Which of the above statements is/are correct?
(a) 1 and 2 only
(b) 2 and 3 only
(c) 1 and 3 only
(d) 1, 2 and 3
Answer: (d)
Explanation: Gokhale‘s Bill of 1911. He made further attempt to draw the attention of the
people of India as well as in England towards the condition of education. On 16th March of
1911, Gokhale presented a Bill in the Legislative Council to make a stronger fight against the
Government. The Bill, however, was more liberal and humble than the resolutions placed
before and the main objective of the bill was to make primary education free and compulsory
in a phased manner. The Bill was basically based on the compulsory Education Acts of
England of 1870 and 1876. Hence statement 1 is correct.
While rejecting Gokhale‘s bill of 1911, the Government promised to extend recurring and
non-recurring grants to primary education as it could not ignore the growing popular demand
for the spread of primary education. The education department had declared the new policy in
the form of Government of India Resolution on February 21, 1913 covering primary,
secondary, higher and women education. Hence statements 2 and 3 are correct.
Source: http://kkhsou.in/main/education/gokhale's_bill.html

32.Consider the Following statements with respect to the Satyagraha Sabha


a)It was initially started by MK Gandhi in South Africa
b)He roped in members from the Home Rule League for this organization
c)It was a form of constitutional protest.
d)All the above

Ans-b

EXPLANATION

In February 1919 Gandhi Ji founded the Satyagraha Sabha to protest against the Rowlatt Act.
He roped in younger members of the Home rule league and Pan islamists.
The Forms of protest finally chosen included observance of a nationwide hartal ,accompanied
by fasting and Prayer and civil disobedience against specific laws ,and courting arrest and
imprisonment.

33.Passive Resistance association was founded by-

a)MK Gandhi
b)DadabhaiNaoroji
c)BalGangadharTilak
d)Annie Besant
Ans-a

EXPLANATION
In 1906 a legislation in South Africa made it compulsory for Indians there to carry at all
times certificates of Registration with their Fingerprints.
The Indians under Gandhi‘s leadership decided not to submit to this discriminatory measure.
Gandhi formed the Passive Resistance Association to conduct the campaign .

34. With reference to the Public Safety Bill of 1928, which of the following statements
is/are correct?

1. The Swarajists were the main opposition for the Bill in the Central Legislative
Assembly.
2. The government was defeated during the passage of this bill.

Select the correct answer using the code given below.


(a) 1 only
(b) 2 only
(c) Both 1 and 2
(d) Neither 1 nor 2

Answer: (c)
Explanation: The Public Safety Bill was aimed at deporting 'undesirable' and 'subversive'
foreigners, who came to India to spread the Communist ideology. British govt believed that
the Communist International sent foreigners to spread communist ideas in India. The
Swarajists defeated the govt on the passage of this bill. As the bill could not be passed as late
as 1929, the British govt arrested several Communist leaders and put them on the Meerut
conspiracy trial.
The Swaraj Party participated in the elections in 1923 and 1926. INC stayed away from the
elctions but supported the work of Swarajists in the legislatures.
Both Statements are correct.
Reference: Page: 240, 244; India's Struggle for Independence, Bipan Chandra and others
Page: 340; A new look at modern Indian history, B.L.Grover and Alka Mehta

35. With reference to Hindustan Republic Association (HRA), which of the following
statements is/are correct?

1. Bhagat Singh and Sukhdev were members of this organisation.


2. It was formed by Chandrashekhar Azad.

Select the correct answer using the code given below.


(a) 1 only
(b) 2 only
(c) Both 1 and 2
(d) Neither 1 nor 2

Answer: (d)
Explanation:Kakori train robbery of 1925 is associated with HRA (Hindustan Republican
Association). HRA was founded in 1924 by RamprasadBismil, JogeshChatterjea,
SachindranathSanyal. Ashfaqullah Khan, RamprasadBismil, RajendraLahiri and Roshan
Singh were hanged while many were sent to Andamans for life. Chandrashekhar Azad
remained at large. Statement 1 is incorrect.
HRA was reorganised as HSRA (Hindustan Socialist Republican Association) under Chandra
Shekhar Azad. Bhagat Singh, Sukhdev and many others were part of this HSRA. Statement
2 is incorrect.
Reference: Page: 299; History of Modern India, Bipan Chandra
Page: 248; India's Struggle for Independence, Bipan Chandra and others

36. What is the correct chronological sequence of the following?


1. Wood's Education Despatch
2. Macaulay's minute on education
3. The Sargent Education Report
4. Indian Education (Hunter Commission)
Select the correct answer using the code given below:
(a) 2, 1, 4, 3
(b) 2, 1, 3, 4
(c) 1, 2, 4, 3
(d) 4, 3, 1, 3

Answer – a

Explanation: Wood‘s Despatch (Magna Carta of English education) – 1854; Macaulay‘s


Minute on Education - 1835; Sargent Education Report – 1944; Hunter Commission – 1882-
83

37. Arrange the following areas containing minerals with the chronological order of
its formation:
1. Coal deposits of Gondwana
2. Diamonds of Vindhayan rocks
3. Granite and mica of Karnataka
4. Tertiary deposits found in Himalayas
Choose the correct answer using the code given below
(a) 1-2-3-4
(b) 2-3-1-4
(c) 3-2-1-4
(d) 2-3-4-1
Answer: c
Explanation: Archean Gneiss and Schist are the oldest rocks formed 4 billion years ago.
Gneiss is Mineral composition varies from granite to gabbros. Schist mostly crystalline
include mica, talc, hornblende, chlorite, etc. Known as the „Basement Complex‟ [They are
the oldest and form the base for new layers].
The diamonds found in the Vindhayan system was formed 1300-600 million years ago.
The Gondwana coal formed in the Permian period about 250 million years ago.
The tertiary deposit of Himalayas was formed around 60-7 million years ago.

38. Consider the following statements:


1. Bulk of the minerals in India belongs to the pre-Palaeozoic age.
2. Non-metallic minerals are both organic and inorganic.
3. Coal is called as mineral fuel as it is part of non-metallic minerals.
Which of the above statements is/are correct?
(a) 1 and 2 only
(b) 2 and 3 only
(c) 1 and 3 only
(d) 1, 2 and 3
Answer: d
Explanation: India is endowed with a rich variety of mineral resources due to its varied
geological structure. Bulk of the valuable minerals is products of pre-palaezoic age and is
mainly associated withmetamorphic and igneous rocks of thepeninsular India. The vast
alluvial plain tractof north India is devoid of minerals of economicuse.
Metallic minerals are further divided into ferrous and non-ferrous metallic minerals. Ferrous,
as you know, refers to iron. All those minerals which have iron content are ferrous such as
iron ore itself and those which do not have iron content are non-ferrous such as copper,
bauxite, etc. Non-metallic minerals are either organic in origin such as fossil fuels also known
as mineral fuels which are derived from the buried animals and plant life such as coal
and petroleum. Other type of non-metallic minerals is inorganic in origin such as mica,
limestone and graphite, etc. Hence the statements are correct.
Source: India People and Economy; NCERT; Chapter 7: Minerals and energy resources

39. Consider the following statements:


1. Hematite is the best quality of iron ore.
2. India is abundant in hematite and siderite types of iron ore.
3. Hematite type is abundant in Odisha whereas Magnetite is abundant in Karnataka.
Which of the above statements is/are correct?
(a) 1 and 2
(b) 2 and 3
(c) 1 and 3
(d) 1, 2 and 3
Answer: c
India is endowed with fairly abundant resources of iron ore. It has the largest reserve of iron
ore in Asia. The two main types of ore found in our country are haematite and magnetite.
Hence statement 2 is incorrect.
It has great demand in international market due to its superior quality. Hematite is the best
quality with 70 per cent metallic content. Hence statement 1 is correct.
Hematite is found in Dharwad and Cuddapah rock systems of the peninsular India. 80 per
cent of haematite reserves are in Odisha, Jharkhand, Chhattisgarh and Andhra Pradesh.
Magnetite has magnetic quality. It has 60-70% metallic content. It is found majorly in
Karnataka region. Hence statement 3 is correct.
Source: India People and Economy; NCERT; Chapter 7: Minerals and energy resources
40. Consider the following statements:
1. Manganese is associated with the Dharwar system of rocks and its occurrence near the
steel industry is an advantage for India being the world‘s largest producer of manganese.
2. It is added as fuel in iron smelting and is involved in manufacturing steel alloys.
3. Odisha has the highest manganese deposits in India.
Which of the above statements is/are correct?
(a) 1 and 2
(b) 2 and 3
(c) 1 and 3
(d) 1, 2 and 3
Answer: d
Explanation: Manganese is an important raw material for smelting of iron ore and also used
for manufacturing ferro alloys. it is mainly associated with Dharwar system. Odisha is the
leading producer of Manganese. Major mines in Odisha are located in the central part of the
iron ore belt of India, particularly in Bonai, Kendujhar, Sundergarh, Gangpur, Koraput,
Kalahandi and Bolangir. Karnataka is another major producer and here the mines are located
in Dharwar, Ballari, Belagavi, North Canara, Chikkmagaluru, Shivamogga, Chitradurg and
Tumkur. Maharashtra is also an important producer of manganese which is mined in Nagpur,
Bhandara and Ratnagiri districts.
Statement 1 is incorrect as the manganese ores are located far from the iron and steel industry
and India is not the largest producer of Manganese in world.Indiahas the second largest
reserve of Manganese.
Source: India People and Economy; NCERT; Chapter 7: Minerals and energy resources

41. Consider the following statements;


1. Bauxite is found in Deccan traps region.
2. It is the ore for aluminium and is found in hydrated aluminium form.
3. Tamil Nadu is the largest producer of bauxite.
Which of the above statements is/are correct?
(a) 1 and 2
(b) 2 only
(c) 1 and 3
(d) 3 only
Answer: b
Explanation: Bauxite is the ore which is used in manufacturing of aluminium. Bauxite is
found mainly in tertiary deposits and is associated with laterite rocks occurring extensively
either on the plateau or hill ranges of peninsular India and also in the coastal tracts of the
country. Hence statement 1 is incorrect. 80 % of bauxite [ore of aluminium] ore is used for
making aluminium, found mainly as hydrated aluminium oxides. Hence statement 2 is
correct.
Odisha happens to be the largest producer of Bauxite. Kalahandi and Sambalpur are the
leading producers. The patlands of Jharkhand in Lohardaga have rich deposits. Gujarat,
Chhattisgarh, Madhya Pradesh and Maharashtra are other major producers. Bhavanagar,
Jamnagar in Gujarat have the major deposits. Tamil nadu, Karnataka and Goa are minor
producers of bauxite. Hence statement 3 is incorrect.
Source: India People and Economy; NCERT; Chapter 7: Minerals and energy resources
42. Consider the following statements:
1. Mica is a metal which has very good insulating capacity used in electrical industry.
2. Mica has shine and has abrasive quality too.
3. India has the largest reserves of mica in the world, and in India Andhra Pradesh has
the highest mica reserves.
Which of the above statements is/are correct?
(a) 1 and 2
(b) 2 and 3
(c) 1 and 3
(d) 1, 2 and 3
Answer: b
Explanation: Mica is a naturally occurring non-metallic mineral that is based on a
collection of silicates. Hence statement 1 is incorrect. Mica is a very good insulator that
has a wide range of applications in electrical and electronics industry. It can withstand high
voltage and has low power loss factor. It is used in toothpaste and cosmetics because of its
glittery appearance. It also acts as a mild abrasive in toothpaste. Hence statement 2 is
correct.India is one of the foremost suppliers of mica to the world. Mica-bearing igneous
rocks occur in AP, Bihar, Jharkhand, Maharashtra, and Rajasthan. Andhra Pradesh is the
leading producer. 1st in production [93 %]. The mica belt lies in Nellore district [Gudur
Mica mines], Vishakhapatnam, West Godavari and Krishna are other important mica
producing districts.
Source: India People and Economy; NCERT; Chapter 7: Minerals and energy resources

43. Consider the following statements:


1. Limestone is found in sedimentary rocks across all the geological ages except the
Gondwana region.
2. Limestone is used in cement industry and iron and steel industry.
3. Dolomite is a type of limestone with magnesium content.
Which of the above statements is/are correct?
(a) 1 and 2
(b) 2 and 3
(c) 1 and 3
(d) 1, 2 and 3
Answer: d
Explanation: Limestone rocks are composed of either calcium carbonate, the double
carbonate of calcium and magnesium, or mixture of both. Limestone also contains small
quantities of silica, alumina, iron oxides, phosphorus and sulphur. Limestone deposits are of
sedimentary origin and exist in all the geological sequences from Pre-Cambrian to
Recent except in Gondwana. 75 per cent Limestone is used in cement industry, 16 per cent
in iron and steel industry [It acts as flux] and 4 per cent in the chemical industries. Rest of
the limestone is used in paper, sugar, fertilizers, etc. Over three-fourths of the total limestone
of India is produced by Madhya Pradesh, Rajasthan, Andhra Pradesh, Gujarat, Chhattisgarh
and Tamil Nadu. Madhya Pradesh is the largest producer of limestone [16 per cent]. Large
deposits occur in the districts of Jabalpur, Satna, Betul, etc. Hence statement 1 and 2 is
correct.
Limestone with more than 10 per cent of magnesium is called dolomite. Hence
statement 3 is correct. When the percentage rises to 45, it is true dolomite. Dolomite is
mainly used as blast furnace flux, as a source of magnesium salts and in fertilizer and glass
industries. Iron and Steel industry is the chief consumer of dolomite [90 per cent] followed by
fertilizer, ferro-alloys and glass. Dolomite is widely distributed in the all parts of the country.
Orissa, Chhattisgarh, Andhra Pradesh, Jharkhand, Rajasthan and Karnataka are the main
producing states and contribute more than 90 per cent of the total production.
Source: India People and Economy; NCERT; Chapter 7: Minerals and energy resources

44. Consider the following statements:


1. Copper is a metal found only in the Vindhyan rock system.
2. It is widely used in electrical industry and is extensively alloyed with aluminium,
nickel, zinc, tin etc.
3. Copper reserves are highest in Madhya Pradesh.
Which of the above statements is/are correct?
(a) 1 and 2
(b) 2 only
(c) 1 and 3
(d) 3 only
Answer: b
Explanation: Copper ore is found in ancient as well as in younger rock formations and
occurs as veins and as bedded deposits. It is found only not in the vindhayan rocks. It is also
found in Jharkhand, Madhya Pradesh etc.Hence statement 1 is incorrect.
Copper is an indispensable metal in the electrical industry for making wires, electric motors,
transformers and generators. It is alloyable, malleable and ductile. It is also mixed with gold
to provide strength to jewellery. It is alloyed with nickel to form stainless steel; with
aluminium to form duralumin; with zinc to form brass; and with tin to form bronze. Hence
statement 2 is correct.
Rajasthan has the highest reserves of copper. Found along the Aravali range.Ajmer, Alwar,
Bhilwara, Chittaurgarh, Dungarpur, Jaipur, Jhunjhunu, Pali, Sikar, Sirohi and Udaipur
districts.Khetri-Singhana belt in Jhunjhunu district is the most important copper producing
area. But the mines in Madhya Pradesh tops in the production with Malanjkhand copper
mines of Balaghat district are the most important ones. Hence statement 3 is incorrect.
Source: India People and Economy; NCERT; Chapter 7: Minerals and energy resources

45. Consider the following statements:


1. Anthracite coal is the best quality coal in the world formed during the carboniferous
era.
2. The carbon content in coal depends on the depth of formation.
3. As the amount of oxygen and nitrogen increase in coal, more is its efficiency.
Which of the above statements is/are correct?
(a) 1 and 2 only
(b) 2 and 3only
(c) 1 and 3 only
(d) 1, 2 and 3
Answer: a
Explanation: Most of the world‘s coal was formed in Carboniferous age [350 million years
ago which is the Best quality coal. Carboniferous age: In terms of absolute time, the
Carboniferous Period began approximately 358.9 million years ago and ended 298.9 million
years ago. Its duration is approximately 60 million years. The name Carboniferous refers to
coal-bearing strata. Anthracite is the type of coal formed during this stage.
Amount of oxygen, nitrogen and moisture content decreases with time while
the proportion of carbon increases [The quantity of carbon doesn‘t increase, only its
proportion increases due to the loss of other elements]. The lesser amounts of oxygen and
nitrogen the more efficiency.Hence statement 3 is incorrect.
Capacity of coal to give energy depends upon the percentage or carbon content [Older the
coal, much more is its carbon content].
Percentage of carbon in coal depends upon the duration and intensity of heat and pressure on
wood. Carbon content also depends on depth of formation. More depth means more
pressure and heat hence better carbon content.
Source: Geology.com

46. Arrange the following types of coal in the decreasing content of carbon:
1. Bituminous
2. Anthracite
3. Peat
4. Lignite
Select the correct answer using the below codes:
(a) 2-1-4-3
(b) 3-4-1-2
(c) 2-1-3-4
(d) 1-2-4-3
Answer: a
Explanation: Anthracite: Anthracite is the highest rank of coal. Unlike other types of coal, it
is usually considered to be a metamorphic rock. It has a carbon content of over 87% on a dry
ash-free basis. Anthracite coal generally has the highest heating value per ton on a mineral-
matter-free basis. It is referred to as hard coal.
Bitumnous coal: Bituminous is the most abundant rank of coal.. Bituminous coal is formed
when a sub bituminous coal is subjected to increased levels of organic metamorphism. It has
a carbon content of between 77 and 87% on a dry ash-free basis and a heating value that is
much higher than lignite or sub bituminous coal. On the basis of volatile content, bituminous
coals are subdivided into low-volatile bituminous, medium-volatile bituminous, and high-
volatile bituminous. Bituminous coal is often referred to as "soft coal".
Sub bituminous coal: it is a lignite that has been subjected to an increased level of organic
metamorphism. This metamorphism has driven off some of the oxygen and hydrogen in the
coal. That loss produces coal with a higher carbon content (71 to 77% on a dry ash-free
basis).
Lignite: is the lowest rank of coal. It is a peat that has been transformed into a rock, and that
rock is a brown-black coal.. It has a carbon content of between 60 and 70% on a dry ash-free
basis. In Europe, Australia, and the UK, some low-level lignites are called "brown coal."
Peat: A mass of recently accumulated to partially carbonized plant debris. Peat is an organic
sediment. Burial, compaction, and coalification will transform it into coal, a rock. It has a
carbon content of less than 60% on a dry ash-free basis.
In the process of transformation (coalification), peat is altered to lignite, lignite is altered to
sub-bituminous, sub-bituminous coal is altered to bituminous coal, and bituminous coal is
altered to anthracite.

Source: geology.com

47. Which of the following coal fields is located in Odisha?


(a) Jharia
(b) Boakro
(c) Raniganj
(d) Talchar
Answer: d
Explanation: The most important Gondwana coal fields of India are located in
DamodarValleyThey lie in Jharkhand-Bengal coal belt and the important coal fields in this
region are Raniganj. Jharia, Bokaro, Giridih, Karanpura. Jharia is the largest coal field
followed by Raniganj. The other river valleys associate with coal are Godavari, Mahanadi
and Sone. The most important coal mining centres are Singrauli in Madhya Pradesh (part of
Singrauli coal field lies in Uttar Pradesh), KorbainChhattisgarh, Talcher and Rampur in
Odisha, Chanda–Wardha, Kamptee and Bander in Maharashtra and Singareni in Telangana
and Pandur in Andhra Pradesh.
Source: India People and Economy; NCERT; Chapter 7: Minerals and energy resources

48. Consider the following statements:


1. Crude petroleum is found in the anticlines and fault traps of sedimentary rocks of
tertiary period.
2. Digboi is an example of field based refinery and Baruni is example of market based
refinery.
3. DigboiNaharkatiya oil field located in Meghalaya was the first oil field in India.
Which of the above statements is/are correct?
(a) 1 and 2
(b) 2 and 3
(c) 1 and 3
(d) 1, 2 and 3
Answer: a
Explanation: Most of the oil reserves in India are associated with anticlines and fault
traps in the sedimentary rock formations of tertiary times. Hence statement 1 is correct. In
Assam, Digboi, Naharkatiya and Moran are important oil producing areas. Hence
statement 3 is incorrect. The major oil fields of Gujarat are Ankaleshwar, Kalol, Mehsana,
Nawagam, Kosamba and Lunej. Mumbai High which lies 160 km off Mumbai was
discovered in 1973 and production commenced in 1976. Oil and natural gas have been found
in exploratory wells in Krishna-Godavari and Kaveri basin on the east coast. Oil extracted
from the wells is crude oil and contains many impurities. It cannot be used directly. It needs
to be refined. There are two types of refineries in India: (a) field based and (b) market based.
Digboi is an example of field based and Barauni is an example of market based refinery.
Hence statement 2 is correct.
Source: India People and Economy; NCERT; Chapter 7: Minerals and energy resources

49. Arrange the following water bodies in the decreasing order of volume on earth:
1. Ice caps and glaciers
2. Oceans
3. Lakes
4. Groundwater
5. Soil moisture
6. Rivers
Select the correct answer using the codes given below:
(a) 2-1-3-6-5-4
(b) 2-1-4-3-5-6
(c) 1-2-4-6-3-5
(d) 1-2-6-4-3-5
Answer: b
Explanation:
Water body Volume in %
Oceans 97.25
Ice caps & glaciers 2.05
Groundwater 0.68
Lakes 0.01
Soil moisture 0.005
Atmosphere 0.001
Streams and river 0.0001
Biosphere 0.00004

Source: Fundamentals of Physical Geography; NCERT; Chapter 13: Oceans

50. Consider the following statements:


1. Continental shelf is the shallowest part of the oceans with gradient less than 1degree.
2. It is a source of fossil fuel.
3. The western continental shelf is narrower than the eastern continent shelf in India.
Which of the above statements is/are correct?
(a) 1 and 2 only
(b) 2 and 3 only
(c) 1 and 3 only
(d) 1, 2 and 3
Answer: a
Explanation: The continental shelf is the extended margin of each continent occupied by
relatively shallow seas and gulfs. It is the shallowest part of the ocean showing an average
gradient of 1° or even less. The shelf typically ends at a very steep slope, called the shelf
break. The width of the continental shelves varies from one ocean to another. The average
width of continental shelves is about 80 km. The depth of the shelves also varies. It may be as
shallow as 30 m in some areas while in some areas it is as deep as 600 m. The continental
shelves are covered with variable thicknesses of sediments brought down by rivers, glaciers,
wind, from the land and distributed by waves and currents. Massive sedimentary deposits
received over a long time by the continental shelves, become the source of fossil fuels.
Statement 3 is incorrect as the western continental shelf is wider than the eastern continental
shelf in India.
Source: Fundamentals of Physical Geography; NCERT; Chapter 13: Oceans

51. Consider the following statements:


1. The continental shelf is the end of the continent.
2. The continental slope is characterised by minor relief features like canyons and
trenches.
Which of the above statements is/are correct?
(a) 1 only
(b) 2 only
(c) Both 1 and 2
(d) Neither 1 nor 2
Answer: b
Explanation: The continental slope connects the continental shelf and the ocean basins. It
begins where the bottom of the continental shelf sharply drops off into a steep slope. The
gradient of the slope region varies between 2-5°. The depth of the slope region varies
between 200 and 3,000 m. The slope boundary indicates the end of the continents. Hence
statement 1 is incorrect. Canyons and trenches are observed in this region, therefore
statement 2 is correct.
Source: Fundamentals of Physical Geography; NCERT; Chapter 13: Oceans

52. Consider the following statements:


1. Trenches are the deepest parts of the world found in the mid oceanic ridge.
2. Deep sea plains are the widest and smoothest part in the whole world.
3. Shelf break is a steep slope at the end of continental shelf.
Which of the above statements is/are correct?
(a) 1 and 2 only
(b) 2 and 3 only
(c) 1 and 3 only
(d) 1, 2 and 3
Answer: b
Explanation: These areas are the deepest parts of the oceans. The trenches are relatively steep
sided, narrow basins. They are some 3-5 km deeper than the surrounding ocean floor. They
occur at the bases of continental slopes and along island arcs and are associated with active
volcanoes and strong earthquakes. Hence statement 1 is incorrect.
Deep sea plains are gently sloping areas of the ocean basins. These are the flattest and
smoothest regions of the world. The depths vary between 3,000 and 6,000m. These plains are
covered with fine-grained sediments like clay and silt. Statement 2 is correct.
The continental shelf typically ends at a very steep slope, called the shelf break. Hence
statement 3 is correct.
Source: Fundamentals of Physical Geography; NCERT; Chapter 13: Oceans

53. Which of the following is not a factor for the spatial temperature variation in the
oceans?
(a) Winds
(b) Latitude
(c) Rate of evaporation
(d) Unequal distribution of land and water
Answer: c
Explanation:Prevailing wind : the winds blowing from the land towards the oceans drive
warm surface water away from the coast resulting in the upwelling of cold water from below.
It results into the longitudinal variation in the temperature. Contrary to this, the onshore
winds pile up warm water near the coast and this raises the temperature
Latitude : the temperature of surface water decreases from the equator towards the poles
because the amount of insolation decreases poleward.
Unequal distribution of land and water: the oceans in the northern hemispherereceive more
heat due to their contact with larger extent of land than the oceans in the southern
hemisphere.
The rate of evaporation in a particular area affects the humidity and precipitation levels in the
ocean rather than a direct effect on the spatial temperature variation of the oceans.
Ocean currents: warm ocean currents raise the temperature in cold areas while the cold
currents decrease the temperature in warm ocean areas.
Source: Fundamentals of Physical Geography; NCERT; Chapter 13: Oceans

54. Consider the following statements:


1. Onshore winds increase the temperature of the sea whereas the offshore winds
decrease the temperature of the sea.
2. Enclosed sea in lower latitudes record higher temperature.
Which of the statements above is/are correct?
(a) 1 only
(b) 2 only
(c) Both 1 and 2
(d) Neither 1 nor 2
Answer; c
Explanation: The winds blowing from the land towards the oceans (off-shore winds-moving
away from the shore) drive warm surface water away from the coast resulting in the
upwelling of cold water from below (This happens near Peruvian Coast in normal years. El-
Nino).
Contrary to this, the onshore winds (winds flowing from oceans into continents) pile up warm
water near the coast and this raises the temperature (This happens near the Peruvian coast
during El Nino event)(In normal years, North-eastern Australia and Western Indonesian
islands see this kind of warm ocean waters due to Walker Cell or Walker Circulation).
The enclosed seas (Marginal Seas – Gulf, Bay etc.) in the low latitudes record relatively
higher temperature than the open seas; whereas the enclosed seas in the high latitudes have
lower temperature than the open seas. Hence both the statements are correct.
Source: Fundamentals of Physical Geography; NCERT; Chapter 13: Oceans

55. Oceans take more time to heat or cool than land because:
(a) They have a very large surface area than earth
(b) Vertical and horizontal mixing of the ocean waters
(c) Water is less denser than land
(d) Temperature changes over the oceans is more prominent than over the land
Answer: b
Explanation; The process of heating and cooling of the oceanic water is slower than land due
to vertical and horizontal mixing of the water by way of ocean currents winds etc. though the
other factors like density of water and surface area is a reason the above stated principle is the
most appropriate explanation.

56. Consider the following statements:


1. Thermocline is the layer in which there is drastic increase in the temperature of the
ocean water.
2. In the polar region the subsurface water is warmer than the surface water.
3. The highest temperature in the ocean is recorded at the equator.
Which of the above statements is/are correct?
(a) 1 and 2 only
(b) 1 and 3 only
(c) 2 only
(d) 3 only
Answer: c
Explanation: boundary region, from where there is a rapid decrease of temperature, is
called the thermocline. About 90 per cent of the total volume of water is found below the
thermocline in the deep ocean. In this zone, temperatures approach 0° C. The second layer
called the thermocline layer lies below the first layer and is characterised by rapid decrease in
temperature with increasing depth. The thermocline is 500 -1,000 m thick. Hence statement
1 is incorrect.
The highest temperature is not recorded at the equator but slightly towards north of it. This is
because at the equator there is perennial precipitation which maintains the balance of
temperature of ocean at that area and is not the hottest part of the ocean. Hence statement 3
is incorrect.
The maximum temperature of the ocean is always recorded at the surface and with increasing
depth the temperature decreases. This is majorly because of the insolation received at the
surface. Contrary to this in the polar region the insolation is very low and the surface level
water is frozen whereas the water below it will be warmer than the surface. Hence statement
2 is correct.
Source: Fundamentals of Physical Geography; NCERT; Chapter 13: Oceans

57. Consider the following statements:


1. Ocean salinity affects the ocean currents
2. The boiling point of saline water is higher than fresh water.
3. Salinity of 32.7 (32.7 o/oo) has been considered as the upper limit to demarcate
‗brackish water‘.
Which of the above statements is/are correct?
(a) 1 and 2 only
(b) 2 and 3 only
(c) 1 and 3 only
(d) 1, 2 and 3
Answer: a
Explanation; Salinity is the term used to define the total content of dissolved salts in sea
water. It is calculated as the amount of salt (in gm) dissolved in 1,000 gm (1 kg) of seawater.
It is usually expressed as parts per thousand or ppt. Salinity of 24.7 (24.7 o/oo) has been
considered as the upper limit to demarcate ‗brackish water‟.
Salinity determines compressibility, thermal expansion, and temperature, and density,
absorption of insolation, evaporation and humidity. The boiling point of the saline water is
higher than the freshwater and the saline water freezes more slowly than saline water. Hence
statement 2 is correct. It also influences the composition and movement of the sea: water
and the distribution of fish and other marine resources. Hence it affects the ocean currents.
Hence statement 1 is correct.
Source: Fundamentals of Physical Geography; NCERT; Chapter 13: Oceans

58. Consider the following statements;


1. Salinity decreases as we go higher up the latitudes.
2. North Sea is more saline than Baltic Sea.
3. The processes of freezing and thawing of ice is also a factor that determines the
salinity of an area.
Which of the above statements is/are correct?
(a) 1 and 2
(b) 2 and 3
(c) 1 and 3
(d) 1, 2 and 3
Answer: d
Explanation: the rate of evaporation determines the salinity. As we move up the latitudes the
insolation and evaporation is lesser than the lower latitudes hence generally the salinity tends
to decrease with increasing latitudes. But local geo-climatic factors determine the exact
salinity of the water body. The North Sea though located on the higher latitude is more saline
because of the north Atlantic drift which is a warm current which makes the water warmer.
The Baltic Sea is less saline because it receives huge inflow of freshwater from rivers. Hence
statement 1 and 2 is correct.
Surface salinity is greatly influenced in coastal regions by the fresh water flow from rivers,
and in Polar Regions by the processes of freezing and thawing of ice. Hence statement 3 is
correct.
Source: Fundamentals of Physical Geography; NCERT; Chapter 13: Oceans

59. Consider the following statements:


1. Large accumulations of water and the flow around them are called Gyres.
2. The strength of the current is referred to as the speed of the current.
3. The deep water currents move faster than the surface water currents.
Which of the above statements is/are correct?
(a) 1 and 2
(b) 2 and 3
(c) 1 and 3
(d) 1, 2 and 3
Answer: a
Explanation: large accumulations of water and the flow around them are called Gyres. These
produce large circular currents in all the ocean basins. Currents are referred to by their
―drift‖. Usually, the currents are strongest near the surface and may attain speeds over
five knots. At depths, currents are generally slow with speeds less than 0.5 knots. The
speed of a current as its ―drift.‖ Hence statement 3 is incorrect. Drift is measured in terms of
knots. The strength of a current refers to the speed of the current. A fast current is considered
strong. A current is usually strongest at the surface and decreases in strength (speed) with
depth. Most currents have speeds less than or equal to 5 knots.
Source: Fundamentals of Physical Geography; NCERT; Chapter 13: Oceans

60. Consider the following pairs:


1. Oyashio current - North Pacific ocean
2. Agulus current - South Atlantic ocean
3. Humbolt current - South Pacfic ocean
4. Mozambique - South atlantic ocean
Which of the above pairs are correctly matched?
(a) 1 and 2 only
(b) 1 and 3 only
(c) 2 and 4 only
(d) 1 and 4 only
Answer: b

61. Which one of the following factors is responsible for the change in the regular
direction of the ocean currents in the Indian ocean?
(a) Indian ocean is half an ocean
(b) Indian ocean has monsoonal drift
(c) Indian ocean is a landlocked ocean
(d) Indian ocean has greater variation in salinity
Answer: b
Explanation: Indian ocean is half an ocean, hence the behavior of the North Indian Ocean
Currents is different from that of Atlantic Ocean Currents or the Pacific Ocean Currents.
Also, monsoon winds in Northern Indian Ocean are peculiar to the region, which directly
influence the ocean surface water movement [North Indian Ocean Currents].
Indian Ocean Currents and Monsoons The currents in the northern portion of the Indian
Ocean change their direction from season to season in response to the seasonal rhythm of
the monsoons. The effect of winds is comparatively more pronounced in the Indian
Ocean.Under the influence of prevailing trade winds [easterly trade winds], the north
equatorial current and the south equatorial current start from the south of Indonesian islands,
moving from east to west.This raises the level of western Indian (south-east of horn of
Africa) ocean by few centimeters. And this creates a counter-equatorial current which
flows between the north equatorial current and the south equatorial current in west-
east direction.The north-east monsoons drive the water along the coast of Bay of Bengal to
circulate in an anti-clockwise direction. Similarly, the water along the coast of Arabian Sea
also circulate in an anti-clockwise circulation.

62. Consider the following statements:


1. Ocean currents are slow-surface movement ofwater in the ocean.
2. Ocean currents assist in maintaining the Earth'sheat balance
3. Ocean currents are set in motion primarily byprevailing winds
4. Ocean currents are affected by the configurationof the ocean
Which of these statements are correct?
(a) 1 and 2
(b) 2, 3 and 4
(c) 1, 3 and 4
(d) 1, 2, 3 and 4

Answer – b

63. Consider the following statements about biofouling:


1. Biofouling refers to an excessive growth of marine animals on structures and ships.
2. Mussels, marine algae and corals are some prominent biofouling agents.
Which of the statements given above is/are correct?
(a) 1 only
(b) 2 only
(c) Both 1 and 2
(d) Neither 1 nor 2
Solution: (c)
Biofouling is an excessive growth of marine animals on structures and ships and is of great
economic nuisance. Hence Statement 1 is correct. Extensive biofouling could weaken
foundations of the harbour and ship hulls. Unabated fouling can make ships heavier and
difficult to manoeuvre. If it happens in the ballast filled with water, it could damage pumps
and other machinery too.
Mussels, especially the Mytilopsis species are the prominent among the list of biofouling
agents. The others are marine algae, microorganisms and corals. Hence Statement 2 is
correct.
https://www.downtoearth.org.in/news/wildlife-biodiversity/dna-test-suggests-mussel-pest-on-
cochin-coast-to-be-invasive-foreigner-62745

64. Consider the following statements:


1. The cultivation of food depends much on pollinators like Bees.
2. Extreme weather events such as droughts and floods increase the population of insects in
tropical countries and helps them thrive.
3. Decline in honey and wild bee populations has been linked to neonicotinoids found in
pesticides.
4. Converting forest land to farmland and resultant biodiversity destruction caused decline in
the population of bees.
Which of the statements given above is/are correct?
(a) 1 and 2
(b) 2 only
(c) 2 and 4
(d) 1, 3 and 4
Solution: (d)
Insects make up the majority of critters living on land. They provide food for birds, pollinate
two-thirds of all crops, replenish soil and keep pests in check. Our planet is home to an
amazing 7 million species of insects and arthropods, which play a crucial role in the food
chain. The food web suffers when insect numbers decrease. This has serious consequences
for the survival of humans.
There are many reasons for the catastrophic fall of pollinators like bees, butterflies, moths
and dung beetles, and other insects that help decompose detritus and faeces. But it‘s mainly
human activities like deforestation, loss of habitats, converting land to farmland and
biodiversity destruction which caused this decline. Hence Statement 4 is correct.
With agriculture comes increased use of chemical fertilizers and pesticides. The worldwide
decline in honey and wild bee populations has been linked to neonicotinoids, which are found
in more that a quarter of all pesticides. Hence Statement 3 is correct.
Climate change driven by human interventions also plays a big role because extreme weather
events such as droughts and floods are fatal to insects as well. Hence Statement 2 is
incorrect.
The food we cultivate depends so much on pollinators like bees. Hence Statement 1 is
correct. It is now well known that industrial-scale intensive agriculture, practised in the
West, and increasingly in India as well, is killing ecosystems. The counterpoint is organic
farming. Studies have found that organic farms harbour far more insects than farms using
chemicals.
https://www.thehindu.com/sci-tech/energy-and-environment/column-the-bees-aint-
okay/article26471282.ece

65. Consider the following statements about Hornbills in India:


1. India is home to nine species of hornbills.
2. They disperse the seeds of many tropical trees and are called 'farmers of the forest'.
Which of the statements given above is/are correct?
(a) 1 only
(b) 2 only
(c) Both 1 and 2
(d) Neither 1 nor 2
Solution: (c)
India is home to nine species of hornbills, of which two are endemic. Hence Statement 1 is
correct.
India is positioned between two bio-geographic realms: the Afrotropical and Indomalayan.
The north-eastern region of India has the highest diversity of Hornbill species (5), though the
number of sympatric species are not as high as in the South-east Asian forests. The Great
Hornbill occurs in north, north-east and south India, apart from Nepal, Bhutan and
Bangladesh.
Hornbills get their name from the horn-like projection called a casque on top of their beak.
They are larger than other forest birds. Hornbills are flashy with their over-sized beaks, bright
skin around their eyes and long eyelashes. Most have a brilliantly coloured pouch of loose
skin at their throat in which they carry fruits, their favourite food.
Hornbills are the 'farmers of the forest' as they disperse the seeds of many tropical trees and
keep the forest alive. Unfortunately, most hornbill species are threatened by habitat loss and
hunting. Hence Statement 2 is correct.

Source: http://www.hornbills.in/about-hornbills.php
http://www.conservationindia.org/articles/hornbill-watch-a-citizen-science-initiative-for-
indian-hornbills

66. Consider the following statements about Canine Distemper Virus (CDV):
1. It has been found to affect lions for the first time, at Gir National Park.
2. There exists no vaccine to safeguard against this virus.
3. CDV causes a highly contagious and life-threatening disease in some wild carnivores
including wolves and tigers.
Which of the statements given above is/are not correct?
(a) 1 only
(b) 2 and 3
(c) 1 and 2
(d) 3 only
Solution: (c)
Canine Distemper Virus (CDV) caused the death of five Asiatic lions in Gir forest, Gujarat,
the Indian Council of Medical Research's (ICMR) National Institute of Virology (NIV) has
found.
In the past, CDV has wiped out 30 per cent of the lion population in the Serengeti forests in
East Africa. Hence Statement 1 is incorrect. Considering the threat posed by CDV, ICMR has
requested the Centre to take immediate steps to save the Asiatic lions, which are heading
towards extinction.
For the first time, NIV has recovered a complete genome of CDV. The sequence was
compared to available CDV sequences and was found to be related to the East African
strains. The scientists at ICMR-NIV recommended the existing CDV vaccine, which should
work as a protective intervention for the Gir lions. Hence Statement 2 is incorrect.
CDV causes a highly contagious and life-threatening disease in dogs. Hence Statement 3 is
correct. CDV also affects other wild carnivores, including wolves, foxes, raccoons, red
pandas, ferrets, hyenas, tigers, and lions.

67. What is meant by the term indicator species?


(a) Species whose physical traits resemble that of an extinct species.
(b) Species whose extinction can lead to the extinction of the entire ecosystem.
(c) species whose abundance and physical traits are used to study ecosystem health.
(d) None of the above
Solution: (c)
Indicator species is a species whose abundance and physical traits are used by scientists to
study forest or ecosystem health.

68. Consider the following about Dholes:


1. They are found predominantly in the forests of the Western Ghats of India.
2. They are classified by the International Union for Conservation of Nature (IUCN) as
‗endangered‘.
Which of the pairs given above is/are correct?
(a) 1 only
(b) 2 only
(c) Both 1 and 2
(d) Neither 1 nor 2
Solution: (c)
Dholes (Cuonalpinus), theIndian wild dogsare classified by the International Union for
Conservation of Nature (IUCN) as ‗endangered‘. Hence Statement 2 is correct.
Faced with threats of forest loss, livestock grazing, and free-ranging dogs, the elusive wild
dogs‘ population (dholes) found predominantly in the forests of the Western Ghats, may be
on a decline. Hence Statement 1 is correct.
https://www.downtoearth.org.in/blog/wildlife-biodiversity/are-dhole-packs-in-peril--63641

69. Consider the following statements about Small Woodbrown butterfly:


1. It is endemic to the eastern Himalayas.
2. They are used as indicator species to study forest or ecosystem health.
Which of the statements given above is/are correct?
(a) 1 only
(b) 2 only
(c) Both 1 and 2
(d) Neither 1 nor 2
Solution: (c)
After a span of 120 long years, researchers at the Sikkim University in Gangtok have
rediscovered the Small Woodbrown butterfly species from Bakhim in Khanchendzonga
National Park.
The Small Woodbrown butterfly, scientifically known as Lethe nicetella is named after its
brown-coloured wings patched with white round spots. It is amongst the smallest members of
the genus Lethe, with wings that are up to 50 millimeters long. The species is endemic to the
eastern Himalayas and occurs in forests lying between elevations of 1,800-2,800 metres.
Hence Statement 1 is correct.
Butterflies are important components of the natural ecosystem. They help to pollinate
flowers—a process which is necessary for plants to produce viable seeds. Also, some species
of butterflies feed on plants, some on animal dung, and very few on smaller insects like
aphids, making them important links of Nature‘s food chain. Butterflies are also used as
indicator species—that is, species whose abundance and physical traits are used by scientists
to study forest or ecosystem health. Hence Statement 2 is correct.
https://www.downtoearth.org.in/news/wildlife-biodiversity/small-woodbrown-butterfly-
rediscovered-in-sikkim-after-120-years-62832

70. Consider the following statements:


1. The Wood Snake is endemic to the Meghamalai Wildlife Sanctuary.
2. It is situated in the Eastern Ghats.
Which of the statements given above is/are correct?
(a) 1 only
(b) 2 only
(c) Both 1 and 2
(d) Neither 1 nor 2
Solution: (a)
A species of wood snake that wasn‘t seen for 140 years has resurfaced in a survey conducted
by scientists in the Meghamalai Wildlife Sanctuary. The species is endemic to the
Meghamalai forests and the Periyar Tiger Reserve landscape.
The snake is a ‗point endemic‘ (found only in Meghamalai). Hence Statement 1 is correct.
Location Districts of Theni and Madurai
Meghamalai has a range of snakes, butterflies and ants. Vaigairiver flows through it. It is a
mountain range situated in the Western Ghats. Hence Statement 2 is incorrect.
https://www.thehindu.com/sci-tech/energy-and-environment/wood-snake-last-seen-in-1878-
rediscovered-by-scientists/article26505577.ece

71. Consider the following statements about Protected Areas (PA):


1. Protected areas are locations which receive protection because of their recognized natural,
ecological or cultural values
2. It does not include Marine Protected Areas and Transboundary Protected Areas
3. Over 5 % of India is designated as protected area.
Which of the statements given above is/are correct?
(a) 1 and 2
(b) 2 only
(c) 2 and 3
(d) 1 and 3
Solution: (d)
Protected areas are those in which human occupation or at least the exploitation of resources
is limited. They receive protection because of their recognized natural, ecological or cultural
values. Hence Statement 1 is correct.
The definition that has been widely accepted across regional and global frameworks has been
provided by the International Union for Conservation of Nature (IUCN) in its categorization
guidelines for protected areas.
There are several kinds of protected areas, which vary by level of protection depending on the
enabling laws of each country or the regulations of the international organizations involved.
The term "protected area" also includes Marine Protected Areas, the boundaries of which will
include some area of ocean, and Transboundary Protected Areas that overlap multiple
countries which remove the borders inside the area for conservation and economic purposes.
Hence Statement 2 is incorrect.

Statement 3 is correct.

Source:
http://wiienvis.nic.in/Database/Protected_Area_854.aspx

72. Consider the following statements about National Park:


1. An area, whether within a sanctuary or not, can be notified by the state government to be
constituted as a National Park
2. Andaman and Nicobar Islands have the largest number of National Parks.
Which of the statements given above is/are correct?
(a) 1 only
(b) 2 only
(c) Both 1 and 2
(d) Neither 1 nor 2
Solution: (a)
An area, whether within a sanctuary or not, can be notified by the state government to be
constituted as a National Park, by reason of its ecological, faunal, floral, geomorphological,
or zoological association or importance, needed to for the purpose of protecting &
propagating or developing wildlife therein or its environment. Hence Statement 1 is correct.
No human activity is permitted inside the national park except for the ones permitted by the
Chief Wildlife Warden of the state

Hence Statement 2 is incorrect.


Source:
http://wiienvis.nic.in/Database/npa_8231.aspx

73. Which among the following are designated as UNESCO World Natural Heritage
sites in India?
1. Great Himalayan National Park
2. Ellora Caves
3. Chilika lake
Select the correct answer using the code given below:
(a) 1 only
(b) 2 and 3 only
(c) 1 and 3 only
(d) 1, 2 and 3
Solution: (a)
The United Nations Educational, Scientific and Cultural Organization (UNESCO) seeks to
encourage the identification, protection and preservation of cultural and natural heritage
around the world considered to be of outstanding value to humanity. This is embodied in an
international treaty called the Convention concerning the Protection of the World Cultural
and Natural Heritage, adopted by UNESCO in 1972.
Source: https://whc.unesco.org/en/about/

74. Consider the following statements about Biosphere Reserves (BRs):


1. They promote sustainable development based on local community efforts and sound
science.
2. Presently, there are 11 notified biosphere reserves in India.
3. They minimize conflict between development and conservation.
Which of the statements given above is/are correct?
(a) 1 only
(b) 1 and 3 only
(c) 2 and 3 only
(d) None
Solution: (b)
The programme of Biosphere Reserve was initiated by UNESCO in 1971. The purpose of the
formation of the biosphere reserve is to conserve in situ all forms of life, along with its
support system, in its totality, so that it could serve as a referral system for monitoring and
evaluating changes in natural ecosystems.
Biosphere reserves are sites established by countries and recognized under UNESCO's Man
and the Biosphere (MAB) Programme to promote sustainable development based on local
community efforts and sound science. Hence Statement 1 is correct.
Presently, there are 18 notified biosphere reserves in India. Hence Statement 2 is incorrect.
Biosphere Reserves (BRs) are representative parts of natural and cultural landscapes
extending over large area of terrestrial or coastal/marine ecosystems or a combination thereof
and representative examples of bio-geographic zones/provinces. The UNESCO has
introduced the designation ‗Biosphere Reserve‘ for natural areas to minimize conflict
between development and conservation. Hence Statement 3 is correct.
BRs are nominated by national government which meet a minimal set of criteria and adhere
to minimal set of conditions for inclusion in the world network of Biosphere reserves under
the Man and Biosphere Reserve Programme of UNESCO.
Tripartite functions of BR (Conservation, Development and logistic support)
 To conserve the diversity and integrity of plants and animals within natural
ecosystems
 To safeguard genetic diversity of species on which their continuing evolution depends
 To ensure sustainable use of natural resources through most appropriate technology
for improvement of economic well-being of the local people
 To provide areas for multi-faceted research and monitoring
 To provide facilities for education and training

75. Which of the following is not a criteria for designation of a Biosphere Reserve?
(a) potential for preservation of traditional tribal or rural modes of living
(b) site that must contain an effectively protected and minimally disturbed core area of value
of nature conservation.
(c) The core area should be typical of a bio-geographical unit and large enough to sustain
viable populations representing all trophic levels
(d) None of the above
Solution: (d)
Criteria for designation of BR
 A site that must contain an effectively protected and minimally disturbed core area of
value of nature conservation.
 The core area should be typical of a bio-geographical unit and large enough to sustain
viable populations representing all trophic levels in the ecosystem.
 The management authority to ensure the involvement/cooperation of local
communities to bring variety of knowledge and experiences to link biodiversity conservation
and socio-economic development while managing and containing the conflicts.
 Areas potential for preservation of traditional tribal or rural modes of living for
harmonious use of environment.
Source: http://www.moef.nic.in/division/biosphere-reserves
76. Consider the following statements:
1. Aldabra atoll has the world's largest population of giant tortoises.
2. It is a world heritage site.
Which of the statements given above is/are correct?
(a) 1 only
(b) 2 only
(c) Both 1 and 2
(d) Neither 1 nor 2
Solution: (c)
Aldabra Atoll is comprised of four large coral islands which enclose a shallow lagoon; the
group of islands is itself surrounded by a coral reef. Due to difficulties of access and the
atoll's isolation, Aldabra has been protected from human influence and thus retains some
152,000 giant tortoises, the world's largest population of this reptile. Hence Statement 1 is
correct.
The World Heritage Centre had taken note of concerns expressed by numerous citizens and
NGOs regarding the recent agreement signed between Seychelles and India to establish a
naval base on Assumption Island, located some 27 km from Aldabra Atoll World Heritage
property (Seychelles). Hence Statement 2 is correct.
https://www.thehindu.com/sci-tech/energy-and-environment/indian-oceans-most-important-
turtle-nesting-site-has-a-plastics-problem/article26592130.ece

77. Consider the following statements about Wildlife Sanctuaries:


1. An area comprising of territorial waters can be notified by the State Government to
constitute as a sanctuary.
2. Maharashtra has the largest number of Wildlife Sanctuaries.
Which of the statements given above is/are correct?
(a) 1 only
(b) 2 only
(c) Both 1 and 2
(d) Neither 1 nor 2
Solution: (d)
Any area other than area comprised with any reserve forest or the territorial waters can be
notified by the State Government to constitute as a sanctuary if such area is of adequate
ecological, faunal, floral, geomorphological, natural. or zoological significance, for the
purpose of protecting, propagating or developing wildlife or its environment. Hence
Statement 1 is incorrect.
Some restricted human activities are allowed inside the Sanctuary area

Hence Statement 2 is incorrect.


Source: http://wiienvis.nic.in/Database/wls_8230.aspx

78. Consider the following statements about Conservation reserves:


1. They are completely owned by the Government of India.
2. These protected area categories are included in the Wildlife Protection Amendment Act
(2003) – The amendment to WPA 1972.
Which of the statements given above is/are correct?
(a) 1 only
(b) 2 only
(c) Both 1 and 2
(d) Neither 1 nor 2
Solution: (c)
Conservation reserves and community reserves in India are terms denoting protected areas of
India which typically act as buffer zones to or connectors and migration corridors between
established national parks, wildlife sanctuaries and reserved and protected forests of India.
Such areas are designated as conservation areas if they are uninhabited and completely
owned by the Government of India, but used for subsistence by communities and designated
as community areas if parts of the land are privately owned. Hence Statement 1 is correct.
These protected area categories were first introduced in the Wildlife (Protection) Amendment
Act of 2002 − the amendment to the Wildlife Protection Act of 1972. These categories were
added because of reduced protection in and around existing or proposed protected areas due
to private ownership of land, and land use. Hence Statement 2 is correct.
https://www.business-standard.com/article/pti-stories/106-coastal-and-marine-sites-
identified-as-conservation-reserves-govt-report-119021301142_1.html

\https://www.thehindu.com/sci-tech/science/protected-areas-matter-to-bumphead-
parrotfish/article26619712.ece

79. Consider the following statements about The Khangchendzonga Biosphere Reserve
(KBR):
1. It is the biosphere reserve with highest altitude in the country.
2. The number of biosphere reserves from India included in the World Network of Biosphere
Reserves is 10.
3. Inclusion of the KBR in the UNESCO list will hit tourism and ongoing research in the
region.
Which of the statements given above is/are not correct?
(a) 1 only
(b) 2 and 3
(c) 1 and 2
(d) 3 only
Solution: (b)
The Khangchendzonga Biosphere Reserve (KBR) of Sikkim, the highest biosphere reserve in
the country that includes the third highest mountain peak in the world, Kanchenjunga (8,586
m), has been included in the UNESCO‘s World Network of Biosphere Reserve (WNBR).
Hence Statement 1 is correct.
With the inclusion of the KBR, one of the highest ecosystems in the world, reaching
elevations of 1,220 m to 8,586 m above sea level, the number of biosphere reserves from the
country included in World Network of Biosphere Reserves has increased to 11.
The last biosphere reserve to be included was the Agasthyamalai Biosphere Reserve in
Kerala in 2016. The Nilgiri Biosphere Reserve was the first reserve from the country to be
included in the WNBR. India has 18 biospheres reserves, of which 11 have been included in
the WNBR. Hence Statement 2 is incorrect.
Inclusion of the KBR in the UNESCO list will boost the unique ecosystem of Sikkim on two
counts: collaborative research and tourism. This development will boost international
research collaboration relating to flora, fauna and ecosystem of the KBR. Hence Statement 3
is incorrect.
https://www.thehindu.com/sci-tech/energy-and-environment/khangchendzonga-biosphere-
reserve-enters-unesco-list/article24659847.ece

80. Consider the following statements about The Important Bird Areas (IBAs)
programme:
1. It is a programme of Birdlife International that aims to identify, monitor and protect a
global network of IBAs.
2. The IBAs serve as conservation areas for protection of birds at the global, regional or sub-
regional level
Which of the statements given above is/are correct?
(a) 1 only
(b) 2 only
(c) Both 1 and 2
(d) Neither 1 nor 2
Solution: (c)
Birds are excellent indicators of ecosystem health. The Important Bird Areas (IBAs)
programme of Birdlife International aims to identify, monitor and protect a global network of
IBAs for conservation of the world's birds and associated ;biodiversity. Hence Statement 1
is correct.
The IBAs serve as conservation areas for protection of birds at the global, regional or sub-
regional level. Hence Statement 2 is correct.
According to Birdlife International, designation of IBAs is based on standardized criteria,
namely
(i) hold significant numbers of one or more globally threatened bird species,
(ii) be one of a set of sites that together hold a suite of restricted-range species or biome-
restricted species and
(iii) have exceptionally large numbers of migratory or congregatory birds.
The IBAs contain a range of habitats, such as wetlands, mudflats, microhabitats in
biodiversity hotspots, grasslands and scrublands, making them excellent indicators of
biodiversity richness. The Bombay Natural History Society (BNHS; www.bnhs.org) and
Birdlife International have identified 467 IBAs in India. Forty percent of these IBAs fall
outside the PA network and thus form an important tool for landscape-level conservation
planning.
Source: http://wiienvis.nic.in/Database/IBA_8463.aspx

81. Consider the following statements about Key Biodiversity Areas (KBAs):
1. It includes areas that contribute to the global persistence of biodiversity.
2. The criteria for designating a site as KBA is given by UN Environment Assembly.
Which of the statements given above is/are correct?
(a) 1 only
(b) 2 only
(c) Both 1 and 2
(d) Neither 1 nor 2
Solution: (a)
Key Biodiversity Areas (KBAs) is an umbrella term commonly used to include areas that
contribute to the global persistence of biodiversity, including vital habitat for threatened plant
and animal species in terrestrial, freshwater and marine ecosystems. Hence Statement 1 is
correct.
Globally KBAs are designated based on 11 criteria defined under five broad categories of
threatened biodiversity; geographically restricted biodiversity; ecological integrity; biological
processes; and, irreplaceability.
The criteria for designating a site as KBA have been described in the document "The Global
Standard for the Identification of Key Biodiversity Areas (2016)" by International Union for
Conservation of Nature (IUCN). IUCN identifies 531 KBA sites in India but these have no
legal basis. Hence Statement 2 is incorrect.

Source:
http://wiienvis.nic.in/Database/Key_Biodiversity_Areas_8647.aspx

82. The Brundtland commission report is related to


(a) UNESCO‘s Man and Biosphere reserve program
(b) Sustainable development
(c) Supplementary report to Lancet‘s report on pollution
(d) None of the above
Answer (b)

83. Consider thefollowing statements with respect to soil erosion


1. Stream bank erosion increase the curvature of rivers (meandering)
2. Gullies impede agriculture and precede formation of ravines
3. Soil erosion started on earth with arrival of man
4. Some form of soil erosion cause death and destruction
Which of the above statements is/are correct?
a. 1 and 2
b. 1, 2 and 3
c. 2, 3 and 4
d. 1,2 and 4.

Answer (d)
Soil erosion due to weathering is a continuous phenomena which has been accelerated due to
impact of man. Landslides area a type of soil erosion that result in death and destruction.

84. Consider the below statements and identify the incorrectoption.


1. The total fresh water supply on earth is less than 3%
2. Majority of this is stored as ground water.
Select the answer using the code given below
a. 1 only
b. 2 only
c. Both 1 and 2
d. Neither 1 nor 2

ANSWER (b)
Majority (~67%) is stored as ice caps.

85. Which of the following is/are the advantages of using biopesticides over chemical
pesticides?
1. Protects water quality
2. Prevents deterioration of soil quality
3. Faster action on pests
Select the correct answer using the code given below.
(a) 2 only
(b) 1 and 2 only
(c) 1 and 3 only
(d) 1, 2 and 3

Answer (B)
Solution:
Chemical pesticides contaminate the ground water thus polluting the primary source of
drinking water. Use of biopesticides prevents water pollution.
Use of chemical pesticides and fertilizers lead to deterioration of soil quality.Biopesticides
prevent this. Slow action on pests is the main characteristic of biopesticides. It is one of the
main disadvantages of biopesticides.

86. Consider the following statements.


1. Central Adoption Resource Authority (CARA) is a statutory body under Ministry of
Women & Child Development.
2. The number of male children placed for in-country adoptions were higher than female
children between 2015 and 2018.
Which of the above statements is/ are correct?
a. 1 only
b. 2 only
c. Both 1 and 2
d. Neither 1 nor 2

Answer: A
Explanation: Statement 1 is correct.
Central Adoption Resource Authority
 Central Adoption Resource Authority (CARA) is a statutory body of Ministry of
Women & Child Development, Government of India.
 It functions as the nodal body for adoption of Indian children and is mandated to
monitor and regulate in-country and inter-country adoptions.
 CARA is designated as the Central Authority to deal with inter-country adoptions in
accordance with the provisions of the Hague Convention on Intercountry Adoption, 1993,
ratified by Government of India in 2003.
 CARA primarily deals with adoption of orphan, abandoned and surrendered children
through its associated /recognised adoption agencies.
Statement 2 is incorrect.India has a skewed gender ratio, but the female child happens to be
the first choice when it comes to adoption.
 The number of female children placed for in-country adoptions and inter-
country adoptions between 2015 and 2018 are relatively higher than male children.
 All the figures put together, female children comprise almost 60% of all in-country
adoptions.
 When it came to inter-country adoptions, the number of female children was even
higher of about 69%.
 According to Central Adoption Resource Authority- the apex regulator of in-country
and inter-country adoptions – the trend is a reflection of declining gender bias in India.

Why this question?


Data from the Ministry of Women and Child Development shows that of the 11,649 children
adopted, 6,962 were girls and 4,687 were boys.

Source: https://www.google.com/amp/s/www.thehindu.com/news/national/60-children-
adopted-in-india-between-2015-and-2018-are-girls/article26241070.ece/amp/

http://cara.nic.in/about/about_cara.html
Refer Pulse Magazine 36 (Feb 10-Feb 16 )

87. Consider the following statements regarding TReDS Platform.


1. It is an online electronic platform for the financing of trade receivables of MSME
Sellers.
2. Receivables Exchange of India Limited (RXIL) has been authorised by Reserve Bank
of India (RBI) to operate the TReDS platform.
Which of the above statements is /are incorrect?
a. 1 only
b. 2 only
c. Both 1 and 2
d. Neither 1 nor 2

Answer: D
Explanation: Both the statements are correct.
 TReDS refers to Trade Receivable Discounting System. TReDS is being setup as per
the RBI guideline issued on December 3, 2014.
 It is an online electronic platform and an institutional mechanism for financing /
factoring of trade receivables of MSME Sellers against Corporate Buyers, Govt. Departments
and PSUs.
 RXIL, a joint venture between National Stock Exchange of India Limited (NSE) and
Small Industries Development Bank of India (SIDBI) has been authorised by Reserve Bank
of India (RBI) to operate the TReDS platform.
 The main objective of the TReDS platform is to address the financing needs of
MSMEs as well as the delayed payments issue.
What are Trade receivables?
Trade receivables are amounts billed by a business to its customers when it delivers goods or
services to them in the ordinary course of business.

Why this question?


State-owned aerospace and defence manufacturer Hindustan Aeronautics (HAL) became the
first public sector enterprise (PSU) to make transaction on RXIL TReDS Platform.

Source: https://www.google.com/amp/s/wap.business-standard.com/article-amp/news-
ians/hal-becomes-first-psu-to-transact-on-treds-rxil-118081701275_1.html

http://www.rxil.in/Faq/Faq

88. Consider the following statements regarding Codex Alimentarius.


1. It is an intergovernmental body of the Food and Agriculture Organization of the
United Nations (FAO) and WHO.
2. It covers processed, semi- processed and raw foods.
Which of the above statements is/ are correct?
a. 1 only
b. 2 only
c. Both 1 and 2
d. Neither 1 nor 2
Answer: C
Explanation: Both the statements are correct.
The Codex Alimentarius is international food standards, guidelines and codes of practice
contribute to the safety, quality and fairness of this international food trade.
Codex standards cover all the main foods, whether processed, semi-processed or raw.
Note:
What is the origin of Codex Alimentarius?
Its name is derived from the Codex AlimentariusAustriacus.CodexAlimentariusAustriacus is
a collection of standards and product descriptions for a variety of foods,voluntary effort of
experts in the food industry and universities established by the Austrian-Hungarian Empire.

Source:
https://www.google.com/amp/s/www.thehindu.com/news/national/kerala/quality-
standards-for-spices-key-to-promoting-trade-says-expert/article26054134.ece/amp/

http://www.fao.org/fao-who-codexalimentarius/about-codex/en/

89. With reference to the Solid Fuel Ducted Ramjet (SFDR) Propulsion Technology ,
Consider the following statements.
1. It is an indigenous technology jointly developed by DRDO and Hindustan
Aeronautics limited (HAL).
2. It helps to propel missiles at supersonic speeds.
3. It will help both surface-to-air and air-to-air missiles to increase their strike range.
Which of the above statements is/ are correct?
1. 1 and 2 only
2. 2 and 3 only
3. 2 only
4. 1 , 2 and 3

Answer: B
Explanation: Statement 1 is incorrect.
Solid Fuel Ducted Ramjet (SFDR): It is a missile propulsion technology jointly developed by
India and Russia.
Both Statements 2 and 3 are correct.
Significance:
 It will help both India‘s surface-to-air and air-to-air missiles to perform better and
enhance their strike range, making them more lethal.
 With it, India can have fastest long-range missiles in two categories, providing full-
fledged and multi-layered aerial protection from hostile attacks.
 Its successful use in missiles will mark India‘s entry into select club of nations that
use next-generation missile technology against manoeuvring targets, compromising
effectiveness of conventional missiles.

What is ramjet?
 Ramjet is a form of air-breathing jet engine that uses the vehicle‘s forward motion to
compress incoming air for combustion without a rotating compressor.
 Fuel is injected in the combustion chamber where it mixes with the hot compressed
air and ignites.
 A ramjet-powered vehicle requires an assisted take-off like a rocket assist to
accelerate it to a speed where it begins to produce thrust.

Source
:https://www.google.com/amp/s/www.thehindubusinessline.com/news/science/india-
successfully-tests-solid-fuel-ducted-ramjet-propelled-missile/article26230574.ece/amp/

Refer Pulse Magazine 36 (Feb 10-Feb 16 )

90. Consider the following statements regarding Customs Convention on International


Transport of Goods under cover of TIR Carnets, 1975 (TIR Convention).
1. It is an international transit system under the support of the United Nations Economic
Commission for Europe (UNECE).
2. The TIR system covers customs transit by road only.
Which of the following statements is /are correct?
a. 1 only
b. 2 only
c. Both 1 and 2
d. Neither 1 nor 2

Answer: A
Explanation: Statement 1 is correct.
 The Customs Convention on International Transport of Goods under cover of
Transports InternationauxRoutiers(TIR )Carnets, 1975 (TIR Convention), is an
international transit system under the auspices of the United Nations Economic Commission
for Europe (UNECE) to facilitate the seamless movement of goods within and amongst the
Parties to the Convention.
 The 1975 convention replaced the TIR Convention of 1959, which itself replaced the
1949 TIR Agreement between a number of European countries.

Statement 2 is incorrect.The TIR system not only covers customs transit by road but a
combination is possible with other modes of transport (e.g., rail, inland waterway, and even
maritime transport), as long as at least one part of the total transport is made by road.

Note :
 India joined the TIR Convention (the United Nations Customs Convention on
International Transport of Goods under cover of TIR Carnets) on 15th June 2017.
 TIR stands for 'Transports InternationauxRoutiers'. The convention allows goods to be
outlined in a TIR carnet and sealed in load compartments.
 Customs verify the carnet and check the seals, with no need for physical checking of
the contents, enabling shipments to pass through countries without being opened at borders.
 The TIR convention will help in fast and easy movement of goods across multiple
countries under a common customs document and guarantee.
 "Reciprocal recognition of customs controls is at the heart of the Convention. This
enables a facilitative and non-intrusive environment for multi-modal transport of goods
through several countries.
Why this question?
The first shipment under the United Nations „Transports InternationauxRoutiers‟ (TIR)
convention arrived in India from Afghanistan through Iran‘s Chabahar Port.

Source: https://www.google.com/amp/s/www.thehindu.com/business/india-gets-first-tir-
shipment-via-chabahar-port-from-afghanistan/article26525062.ece/amp/

http://www.ficci.in/pressrelease-page.asp?nid=3375

91. Global Human Capital Index is released by


a. International Labour Organization
b. World Bank
c. World Economic Forum
d. United Nations Development Programme

Answer: B
Explanation: Human Capital Index (HCI) is been brought out by World Bank, which is first
of its kind index.
 The HCI is based on five parameters child survival, school enrolment, quality of
learning in schools, healthy and safe environment for growth, and adult survival for each of
the 157 countries it mapped.
 These parameters were chosen because studies show that they are closely linked with
a population‘s productivity.
 For each country, the HCI value ranges between 0 and 1 and shows how far below its
potential a country performs.

Source :http://pib.nic.in/newsite/PrintRelease.aspx?relid=184131

92. Consider the following with regards to Astrobatrachuskurichiyana frogs.


1. It is one rare species native to the islands of Andaman and Nicobar .
2. The species has been named Astrobatrachuskurichiyana for its constellation-like
markings and the Indigenous Tribes living on the hill range where it is found.
Which of the above statements is\are correct?
a. 1 only
b. 2 only
c. Both 1 and 2
d. Neither 1 nor 2

Answer:B
Explanation:Statement 1 is incorrect.It has been found in wayanad district of Kerala.
Statement 2 is correct.The species has been named Astrobatrachuskurichiyana for its
constellation-like markings and the indigenous people of Kurichiyarmala, the hill range
where it was found. It is dark brown with a bright orange underbelly, and speckled with pale
blue dots.
Kurichiya Tribe
 The tribe is also known as Malai Brahmins or Hill Brahmins.Theyare the second
largest adivasi community in Wayanad district.
 They stand at the top of the caste hierarchy among the hill tribes of Wayanad.The
community was named Kurichiya by the Kottayam Raja for the community‘s expertise in
archery.
 The name is derived from the phrase ‗kurivechavan‘, which means ‗he who took aim‘.

Source: https://indianexpress.com/article/explained/astrobatrachus-kurichiyana-frog-
species-millions-of-years-old-newly-found-in-ghats-5627136/

Refer Pulse Magazine 36 (Feb 10-Feb 16 )

93. Consider the following statements about the Mega Food Park Scheme.
1. The Ministry of Agriculture and Farmers Welfare is implementing Mega Food Park
Scheme in the country.
2. India has only two Mega Food Park in the country.
3. India‘s first mega food park ‗Srini Mega Food Park‘, was opened in Chittoor in
Andhra Pradesh in 2012.
Which of the above statements is\are correct?
a. 1 and 2 only
b. 2 and 3 only
c. 3 only
d. 1 , 2 and 3
Answer:C
Explanation:Both the Statement 1 and statement 2 are incorrect.To give a major boost to
the food processing sector by adding value and reducing food wastage at each stage of the
supply chain with particular focus on perishables, the Ministry of Food Processing Industries
is implementing Mega Food Park Scheme in the country. So far there are Twelve Mega Food
Parks.
Statement 3 is correct.
Note:The Scheme of Mega Food Park aims at providing a mechanism to link agricultural
production to the market by bringing together farmers, processors and retailers so as to ensure
maximizing value addition, minimizing wastage, increasing farmers income and creating
employment opportunities particularly in rural sector. The Mega Food Park Scheme is based
on ―Cluster‖ approach and envisages creation of state of art support infrastructure in a well-
defined agri / horticultural zone for setting up of modern food processing units in the
industrial plots provided in the park with well-established supply chain.

Source:
http://mofpi.nic.in/Schemes/mega-food-parks

Refer Pulse Magazine 37 (Feb 17-Feb 23 )

94. Consider the following statements about Agri-Market infrastructure fund.


1. AMIF will be created with Ministry of Agriculture and Farmers Welfare.
2. AMIF will provide the State/UT Governments subsidized loan for developing
marketing infrastructure in Agriculture Produce Market Committees (APMCs) and Grameen
Agricultural Markets (GrAMs).
3. In these GrAMs, physical and basic infrastructure will be strengthened using
MGNREGA and other Government Schemes.
Select the correct answer using the codes given
a. 1 and 3 only
b. 2 only
c. 1 , 2 and 3
d. 2 and 3 only
Answer:D
Explanation:Statement 1 is incorrect.The fund will be created with NABARD for
development and up-gradation of agricultural marketing infrastructure in Gramin Agricultural
Markets and Regulated Wholesale Markets.
Statement 2 and 3 are correct.AMIF will provide the State/UT Governments subsidized
loan for their proposal for developing marketing infrastructure in 585 Agriculture Produce
Market Committees (APMCs) and 10,000 Grameen Agricultural Markets (GrAMs). States
may also access AMIF for innovative integrated market infrastructure projects including Hub
and Spoke mode and in Public Private Partnership mode. In these GrAMs, physical and basic
infrastructure will be strengthened using MGNREGA and other Government Schemes.

Source: http://pib.nic.in/newsite/PrintRelease.aspx?relid=188133

Refer Pulse Magazine 36 (Feb 10-Feb 16 )

95. Which of the following statements is \are the correct regarding Swachh Shakti
Programme.
1. It aims to provide separate toilets for girls in all government schools.
2. It aims at behavioural change by providing monetary assistance to women households
for constructing toilets.
3. It aims to bring to focus the leadership role played by rural women in the Swachh
Bharat Mission.

Select the correct answer using the codes given


a. 1 and 2 only
b. 2 and 3 only
c. 3 only
d. 1 , 2 and 3
Answer:C
Explanation:Both the statements 1 and 2 are incorrect.
 The Swachh Shakti Programme is a national event which aims to bring in to focus the
leadership role played by rural women in Swachh Bharat Mission.
 Launched in 2017, the Programme is a part of ongoing activities under the aegis of the
Swachh Bharat Mission, launched on October 2, 2014 by the Prime Minister NarendraModi
to achieve a clean and Open Defecation Free (ODF) India by October 2, 2019.
 The programme is attended by women sarpanchs from across the country.The first
edition of Swachh Shakti programme was launched from Gandhinagar, Gujarat by the Prime
Minister NarendraModi on International Women‘s Day 2017. Statement 3 is correct.

Source: http://pib.nic.in/PressReleaseIframePage.aspx?PRID=1563870

Refer Pulse Magazine 36 (Feb 10-Feb 16 )

96. Sunshine Policy recently in news is related to,


a. South Korea‘s foreign policy towards North Korea.
b. China‘s resolution to end 2 child policy to revive their aging population.
c. India‘s roadmap in dealing with the Food Processing Industry.
d. NASA‘s space mission to explore the unexplored parts of Sun.

Answer:A
Explanation:
 The Sunshine Policy refers to the theoretical basis for South Korea's foreign policy
towards North Korea.The formal name of the policy was the "Comprehensive Engagement
Policy towards North Korea" and first announced by the 15th South Korean President, Kim
Dae-jung, in 1998.
 The policy emerged largely in the context of growing economic gap between the two
Koreas, where the South was moving in the path of strengthening its nation powered by the
economic prosperity achieved from president Park Chung-hee's administration in the 1970s
throughout the 1990s while the North was falling into severe economic decline.
 Facing bankruptcy and spending excessive portion of its funds on warfare along with
the nuclear program, North Korea faced widespread starvation among its people during the
time.Sunshine Policy was aimed at mitigating this gap in economic power and restoring lost
communication between two nations.
The Sunshine policy was based on three following principles:
 No armed provocation by the North will be tolerated.
 The South will not attempt to absorb the North in any way.
 The South actively seeks cooperation.
Source: http://large.stanford.edu/courses/2017/ph241/min2/
https://en.wikipedia.org/wiki/Sunshine_Policy

97. Consider the following pairs. Which of these is\are correctly matched.
Islands : Countries
1. Reunion : Greece
2. Corsica : France
3. Chagos Archipelago : United Kingdom
Select the correct answer using the codes given.
a. 1 and 2 only
b. 2 and 3 only
c. 3 only
d. 1 and 3 only
Answer:B
Explanation:
 The Chagos Archipelago is a group of seven atolls comprising more than 60
individual tropical islands in the Indian Ocean.
 It is officially part of the British Indian Ocean Territory. Reunion is an overseas
department and region of France and an island in the Indian Ocean.
 Corsica is an island in the Mediterranean Sea and one of the 18 regions of France.
 Recently, The International Court of Justice (ICJ) issued an advisory opinion that
Britain has an obligation to end its administration of the Chagos Archipelago — home to the
U.S. military base of Diego Garcia — and complete the process of decolonisation of
Mauritius.

Source: https://www.thehindu.com/news/international/uk-should-hand-over-chagos-
islands-to-mauritius-icj/article26367827.ece
98. With reference to the scheme,“SevaBhojYojna” launched by the Government of
India, consider the following statement
1. It aims to reimburse Central Government share of Central Goods and Services Tax
(CGST) and Integrated Goods and Service Tax (IGST) of Charitable Religious Institutions
(CRIs) providing food.
2. It is an initiative of the Union Ministry of Food Processing Industries.
Which of the following statements given is/are correct?
a. 1 only
b. 2 only
c. Both 1 and 2
d. Neither 1 nor 2

Answer:A
Explanation: Statement 1 is correct. The scheme envisages to reimburse the Central
Government share of Central Goods and Services Tax (CGST) and Integrated Goods and
Service Tax (IGST)so as to lessen the financial burden of such Charitable Religious
Institutions who provide Food/Prasad/Langar (Community Kitchen)/Bhandara free of cost
without any discrimination to Public/Devotees.
Statement 2 is Incorrect.It is an initiative of the Ministry of Culture.

Source: http://vikaspedia.in/social-welfare/minority-welfare-1/seva-bhoj-yojna

99. Consider the following statements about Global Footprint Network.


1. Global Footprint Network‘s goal is to create a future where all humans can live well,
within the means of one planet Earth.
2. Global Footprint Network calculates Earth Overshoot Day every year.
Which of the above statements is/are correct?
a. 1 only
b. 2 only
c. Both 1 and 2
d. Neither 1 nor 2

Answer:C
Explanation:Both the statements are correct.The Global Footprint Network is an
international think tank promoting and driving sustainable policy decisions in a world of
limited resources. Together with its various partners, Global Footprint Network (GFN)
coordinates research, develops methodological standards, and provides decision makers with
a menu of tools to help the human economy operate within Earth‘s ecological limits.
 Global Footprint Network calculates Earth Overshoot Day every year using
Ecological Footprint accounting, which adds up all of people‘s competing demands on
nature, including demand for food, timber, and fibers (cotton); absorption of carbon
emissions from burning fossil fuels; and buildings, roads and other infrastructure.
 August 1 is the earliest Earth Overshoot Day since the world went into ecological
overshoot in the 1970s.The Global Footprint Network was founded in 2003 with the aim to
safeguard a sustainable world for all people in which they can live a fulfilled life within
natural ecological boundaries.
 The headquarters of the Global Footprint Network is located in Oakland (USA), with
further offices in Switzerland, Belgium and the Philippines.

Earth Overshoot Day


Earth Overshoot Day (EOD) means the day when human consumption exceeds the earth‘s
capacity in that particular year to regenerate natural resources.It is the day the world
completely consumes all the natural resources produced that year.It is calculated by Global
Footprint Network.

Source: https://www.thehindu.com/sci-tech/energy-and-environment/an-end-to-our-
means/article24531989.ece

100. Consider the following statements about Damodar River


1. Damodar occupies the western margins of the Meghalaya Plateau.
2. It flows through a rift valley.
3. The Barakar River is the main tributary of the Damodar River
Select the correct answer using the given codes.
a. 1 and 3 only
b. 2 only
c. 2 and 3 only
d. 1, 2 and 3
Answer: C
Explanation:Damodar River, river in northeastern India, rising with its many tributaries,
notably the Bokaro and Konar, in the Chota Nagpur plateau of south-central Bihar state. It
follows a generally eastward course for 368 miles (592 km) through West Bengal to join the
Hugli (Hooghly) River southwest of Kolkata(Calcutta).

Das könnte Ihnen auch gefallen